Sei sulla pagina 1di 665
Quantitative Aptitude for Competitive Examinations WRERY - WY AW ‘The aim of this publication is to supply information taken from sources believed to be valid and reliable, This is not an attempt to render any type of professional advice or analysis, nor is it to be treated as such. While much care has been taken to ensure the veracity and currency of the formation presented within, neither the publisher nor its authors bear any responsibility for any damage arising from inadvertent omissions, negligence or inaccuracies (typographical or factual) that may have found their way ito this book. Copyright © 2008 Dorling Kindersley (India) Pst. Lid. ‘This book is sold subject to the condition that it shall not, by way’of trade or otherwise, be lent, resold, hired out, or otherwise citeulated without the publisher's prior writen consent in any form of binding or cover other than that in which itis published and without a similar condition including this concition being imposed on the subsequent purchaser and without limiting the rights under copyright reserved above, no pact of this publication may be reproduced, stored in of introduced into a retrieval system, er transmitted in any form or by any means (electronic, mechanical, photocopying, recording or otferwise), without the prior written permission of both the copyright ‘owner and the above-mentioned publisher of this book.. ISBN 978-81-317-1956-5 First Impression Published by Dorling Kindersley (India) Pvt. L1d., licensees of Pearson Education in South Ati Head Office: 482 FIE, Patpargarj. Delhi 110 092, india, Registered Office: 14 Local Shopping Centre, Panchsheet Park. New Delhi 110 017. India Laser typeset by Add Computers, Delhi. Printed in India by Anubha Printers. Preface L Numbers 2. HCP and L.CM. of Numbers 3, Square Root and Cube Root 4. Simplification 5. Percentage 6 Average 7. Ratio, Proportion and Variation & Partnership 9. Profit, Loss and Discount 10. Time and Work U1 Pipes and Cisterns 12 Time, Speed and Distance 13. Boats and Streams 14 Races atid Games of Skill 15. Mixtures and Alligation 16. Problems on Ages Simple Interest Compound Interest and Debentures Clocks and Calendar Progressions Permutations and Combinations Probability ‘Mensuration 1: Area and Perimeter RRRBRER SES Mensuration I: Volume and Surface Arca 7. Plane Geometry Set Theory Linear Equations Quadratic Equations Polynomials eSeRsR Contents List 21-218 313.12 ALAI8 S1-534 6.1-6.26 71-732 818d 9.1-9.32 10.1-1033 Hat16 124-12.44 13.1-18.12 11-1410 151-1520 16.1-16.10 171-1720 181-1820 19,119.12 ool 211-2114 22.1-22.32 2123.33 241-2433 25.1-25.68 26.1-2648 3LI31.14 aa You have either reached a page that is unavailable for viewing or reached your viewing limit for this book. In Hindu Arabic System, we use ten symbols 0, 1, 2.3. 4,5, 6, 7, , 9 called digits to represent any number. This is the decimal system where we use the numbers 0 te 9. Oiis called insignificant digit whereas 1, 2,3, 4,5. 6, 7. 8,9 are called significant digits A group of figures, denoting a number is called ‘numeral. For a given numeral, we start from extreme right as Unit’s place, Ten’s place, Hundred’s place and so on. Mlustration 1. We represent the number 309872546 as shown below: | Ten Crore 108 14] Ten Thousand 104 [=| Crores 107 4] Thousend 10° >| Hundred 102 > Units 10° | Lacs 105 We read it as “Thirty crores, ninety eight lakh thousands five hundred and forty six. * Iw this numeral The place value of 6 is 6 * 1 = 6 The place value of 4 is 4 * 10 = 40 ‘The place valve of 5 is 5 * 100 = S00 The place value of 2 is 2 = 1000 = 2000 and so on, The face value of a digit in a number is the value itself wherever it may be. seventy two Thus, the face value of 7 in the above numeral is 7. ‘The face value of 6 in the above numeral is 6 and so on. fies EN Natural Numbers Counting numbers 1, 2, 3.4. 5, numbers. The set of all natural numbers can be represented by N= {1,2,3,4,5, 0} are known as natural REVIEW OF FUNDAMENTAL CONCEPTS Whole Numbers If we include 0 among the natural numbers, then the numbers 0, 1,2, 3, 4, 5, .. are called whole numbers. ‘The set of whole numbers can be represented by W= (0, 1,2,3,4, 55) Clearly, every natural number is a whole number but is a whole number which is not a natural number. Meas All counting numbers and their negatives including zero are known as integers. ‘The set of integers can be represented by Zor 4,3, -2.-1,0,1, 2,3, 4, Positive Integers The set J" = {1, 2, 3, 4 aud is the set of all positive imegers. Clearly, positive integers and natural numbers are synonyms, Negative Integers ‘The set imegers. 0 is neither positive nor negative. 1, -2,-3, ..} is the set of all negative Non-negative Integers The set (0, 1, 2,3, } is the set of all non-negative imvegers. Rational Numbers ‘The numbers of the form £, where p and q are integers 4 43 and q # 0, are known as rational numbers, e. =5,2) 2 ais, 8 I ‘The set of all rational numbers is denoted by Q. aa You have either reached a page that is unavailable for viewing or reached your viewing limit for this book. aa You have either reached a page that is unavailable for viewing or reached your viewing limit for this book. aa You have either reached a page that is unavailable for viewing or reached your viewing limit for this book. waster a1- ows dus 2 = 97425 (©) 4536 «35 = +(4563 «70)= 4319410) 159705 ‘SC 4. Multiplication of a given number by 5, 25, 125, 623, tc., that is, by a number which is some power of 5. Method: Place as many zeros to the right of the ‘multiplicant as is the power of 5 in the multiplier, then divide the mumber so obtained by 2 raised to the same power as is the power of 5. Mustration 6 Mi (a) 39825 (b) 4739 « 25 (©) 7894 * 125 (d) 4863 « 625 ition (a) 3982 « 2= 282? ~ 9910 (0) 4739 % 25 = 473900 473900 - iyggrs (e) 7994 175 = 7824000 _ 7894000 2 8 = 986750 {4863 «6x5 = 28620000 _ 48630000 3 16 = 3039375 Distributive Laws For any three numbers a, b, c, we have (@ axbtarvcnax(b+e) (b) axb-axc=ax(b-o) Mlustration 7 438 « 637 + 438 x 367=? Soluition 438 « 637 +438 « 367 = 438 » (637 + 367) = 430 1000= 438000 Mlustration 8 674 x 832 - 674 x 632 ‘Solution: 674 « 832 - 674 » 632 = 674 » (832 ~ 632) = 674 « 200= 134800 ‘SQUARES (SHORT-CUT METHODS) SC 1. To square any number ending with 5 Method: _(A5y Mlustration 9 (@) @5? (6) (857 =8 (8 + 1/25 = 72/25 = 7225 Numbers 1.5 SC 2. To square a number in which every digit is one ‘Method: Count the number of digits in the given number and start writing numbers in ascending order from one to this number and then in descending order up to one. lustration 10 (a) P= 11 (e) 111? = 1234321 (4) 222 = 2? (111)? = 4 (12321) ~ 49284 (e) 3333? = 37 (11? = 9 (1234321) = T110RBRO (b) 11? = 12328 SC 3. To square a number which is nearer (0 Idk. Method: Use the formula: te +e (ete) ty Mustration 11 97 +3) (97-3) +3? 400+ 9 = 9409 (b) (1027? = (102 ~ 2) (102 + 2)+ 2? = 10400 + 4 = 10404 (c) (994? = (994 + 6) (994 - 6)+ 6 988000 + 36 = 988036 (d) (100: (1005 ~ 5) (1005 + 5) +5? = 1010000 + 25 - 1010025 Tale Multiply the integral portion by the next higher Se Syunesar 1 integer and add " 4 iy 1 1 For example, (54) -5~6+ 4-304 ° (+4) 44 SC $. Square of a number ending in 25 Muliply the hundred’s digit by a number corsisting of the hundred's digit with 2 5 10 its right and attach 625 10 the right of the product For example, (425° :4 «45 = 180 Hence, square of 425 is 180625 SC 6. Finding squares, cubes and higher powers of a number consisitiag of nines only. (@) Square: If m is the number of nines in the number, write (n ~ 1) nines followed by 8 and then (n ~ 1) zeros followed by 1 ©-g,(99997 = 99980001 aa You have either reached a page that is unavailable for viewing or reached your viewing limit for this book. aa You have either reached a page that is unavailable for viewing or reached your viewing limit for this book. aa You have either reached a page that is unavailable for viewing or reached your viewing limit for this book. SCRA number oa being divided by d, and d, successively leaves the remainders 7, and r,, respectively. If the number is divided by d « then the remainder is = (d, « r, + 7). Wlustration 21 A number on being divided by 10 ard U1 successively leaves the remainders 5 and 7, respectively. Find the temainder when the same munaberis divided by 110. ‘Sollition, The required remainder yX tytn = 1OXT+5=75 SC 4 To find the number of numbers divisible by a certain integer. ‘The method is best illustrated with the help of following example. Mustration 22 How mary numbers up to 532 are divisible by 157 Solution We divide 532 by 15 532-35 41547 The quotient obiained is the required number of umbers. Thus, there are 38 such numbers. Mlustration 23 How many numbers up to 300 are divisible by $ and 7 together? Solution 1.CM. of 5 and7=35 We divide 300 by 35 300 = § x 35 +20 Thus, there are 8 such numbers. SC 5 Two numbers when divided by a certain divisor give remainders r, and r,. When their sum is divided by the same divisor, the remainder is r The divisor is given by ry + rs ~ Fy. Mlustration 24 Two numbers when divided by a certain divisor give remainders 473 and 298, respectively. When their sum is divided by the same divisor, the remainder is 236, Find the divisor. Solution The required divisor 37 + 298 -236= 490 ‘A number system is nothing more than a code. For each distinct quantity there is an assigned symbol. The most familiar number system is the decimal system which uses 10 cigits, that is, 0, 1, 2, 3, 4. 5, 6, 7, 8, 9. The main adventage of this system is its simplicity and long use. Most of the ancient societies used this system. Even in ‘our everyday life we use this system and is sometimes being taken as the natural way to count. Since this Numbers 19 system uses 10 digits it is called a system to base 10, ‘A binary number system is a code that uses only two basic symbols, that is, 0 and 1. This system is very useful in camputers. Since, in this system, only two symbols are there, it ean be used in electronic industry using ‘on’ and “oft” positions of a switch denoted by the two digits 0 and 1 Decimal Number System Decimal number system used 10 digits, 0 through 9, that is, the digits 0, 1, 2,3, 4,5, 6, 7,8 9 Binary Number System Binary means two. The binary number system uses only two digits, O and 1 Base or Radix The base or radix of a number system is equal to the number of digits or symbols used in that number system. Forexample, decimal system uses 10 digits, so that base of decimal system (that is, decimal numbers) is 10. Binary numbers have base 2. ‘A subscript attached to a number indicates the base of the number. For example, 100, means binary 100. 100, stands for decimal 100. Weights In any namber toa given base, each digit, depending on its position in the number has a weight in powers of the base. Mlustration 25 In the number ($342), The weight of 2 is x” ‘The weight of 4 is x! The weight of 3 is x7 The weight of 5 is. ‘The sum of al the digits multiplied by their respective ‘weights is equal to the decimal equivalent of that number and gives the total amount represented by that number (5342), = Gr +3844 209 iMustration 26 5 7 0 3 4 10 10% 10 to! 1? Number to the base 10, that is, decimal number weights 2 $x 10847 «103+ 0% 10? +3 «10+ 4» 10" /alue represented or decimal equivalent Illustration 27 1 1 © @ 1 Number to the base 2 2 PP 2! 2 thatis, binary number weights Demat x son r2sox ae x 0 6+8+1=25 = Decimal equivalent or value represented by. 001 aa You have either reached a page that is unavailable for viewing or reached your viewing limit for this book. aa You have either reached a page that is unavailable for viewing or reached your viewing limit for this book. aa You have either reached a page that is unavailable for viewing or reached your viewing limit for this book. 2 # (4x7) Step 2. Multiply 7 by 2 and | by 4, crosswise, and add these two products together, i., 7 * 2 = 14 and 1x = 4; 14+ 4= 18; 18 +2 = 20, Write down 0 and carry over 2 24 17 2028 (x71 x4 42 ‘Multiply the two left-hand most digits, vertically, that i, 1 2= 2; 2 +2 (camy digi) = 4. Write down 4 as the left-hand most digit to give the final answer 408 24 Step 3. Ly 408 (@x1)+2 Note: Th: above method can be started either from the right or from the left Diagrammatic Representation of the Working Rule Step 3 Step 2 Step | °° <2 ° 6 ° é ‘One-step multiplication: 204 24 17 or Lx2Px 71x afrxd 40 8 1/30/,8 oF 408 Exercises Find the following products: 1, 48 «23, Lx 65 3. 1683 4. 81x94 5126 48, 6. 84.67 Answers 1, 04 2 210 3, 6208 4, 1614 5, 516 6, 5628 -Mustration 39 Multiplication of 427 ané 32. Step 1. Multiply 2 by 7 to get 14. Write down 4 and carry over 1 Numbers 1.13 42 3 if QxD Multiply 2 by 2 and 3 by 7. Add the two products and then add I (cary over) to get 26. Write down 6 and carry over 2 ‘step2. 427 32 264 (2e243x7)41 Step3. Multiply 4 by 2 and 2 by 3. Add the two prod- ucts and then add 2 (carry over) to get 16. Write down 6 and carry over 1 424 "a 32 1626 4 (2x4+2%3)42 Step 4. Multiply 4 by 3 and add 1 (carry over) 10 get 13, The final answer is 13664. 427 32 Thee (4*3)+1 Diagrammatic Representation Step4 Step} Step2_——_ Step! e00 900 000 © ©000 Me dhl One-step multiplication: 427 32 43D A+ 3 2/2 243x727 B 6 4 or 427 32 136,64 or 13664 Exercises Find the following products. 1, 2867 x 42 2, 563435 3, 7826 46 4. 6043 » 57 3. 8024%89 6, 7053 x 32 aa You have either reached a page that is unavailable for viewing or reached your viewing limit for this book. aa You have either reached a page that is unavailable for viewing or reached your viewing limit for this book. aa You have either reached a page that is unavailable for viewing or reached your viewing limit for this book. IMlustration 54 Obiain the following products 1 (x tye + 99) 2. (2r~ 3yKax + 1) 3. (2x7 + 3x +4) + 5x +3) 4. (ct + 2x3 + Be? + ae + 2x4 + ae $x? + 2 + 3), Solution 1. x+y x#9y + May +9? 2 detap 4x + Ty 8x7 + 26xy +2? 3B owdsavtd 3x7 + 5x43 6x4 + 19) +3327 + 29x + 12 4 428432 tae 42 att as +s? +20 +3 eS BT 1S! + 24x + 304 + 2dr! + 1? + 16+ 6 CAUTION If and when a power of x is absent, it should be a zero coefficient and the multiplication should be carried on exactly as before. Mlustration 55 Obtain the product (48 +3142) (70) + 2x? + 4), Solution We can write: 4x’ + ox? + 3x+2 Jatt Bst + ox + 4 Dae = BO + 21a + 36x Fa + IDeA 8 Exercises Obtain the following products: 1. G+ 2yjox + ay) 2. (x +B)Gx— dy) 3. (2 + Bet 3YQx2 + Ae #7) 4, (ax) + x + 520 +37 = 7) 5 S438 + Dal + Suh y+ Oe ae +e 420-43) Answers 13x? + Lay + 8y? 2. 3a? + 17ay— 28)? 3. 2x + Bx) + 152 + 26x +21 4. 2x! + 6x9 + 8x9 + 25x) + 21x! + 39x5 + 32x +203 + 23x + Ix +6 emekoa nea There are a number of methods of finding the square of 1a number. The method we shall discuss here gives a general Number @ 1.17 procedure of finding square of any number and is based on a process known as Dvandva Yoga or the dupler combination process. The term Drandva Yoga or duplex is used in two different senses. The first one is by squaring; and the second one is by cross-muliplication. In the present context, itis used in both the senses (a and 2ab). Duplex of a Number For a single-digit number, duplex is its square. For example, duplex of 5 is 5? = 25, duplex of 7 is 7? = 49 Fora two-digit mimber, duplex is twive the product of the evo digits, For example, duplex of 43 is D =2 x (4 * 3)= 24 Duplex of 62 is D=2 «(6 x2) = 24 Duplex of 76 is D=2 (7 x 6) = 84 For a three-digit number, duplex is twice the product of the outer vo digits plus the square of the middle digit, For example, Duplex of 352 is D= 2 «(3x 2)+ 5? =37 Duplex of 534 is D= 2 x (5x 4)+ 7 =49 Duplex of 736 is D = 2 « (7 * 6) + 3? = 93 For a four-digit number, duplex is the sum of two products: first, twice the product of the extreme digits, and second, twice the product of the middle two digits. For example, Duplex of 2345 isD=2 (25) 42% (3 4)—44 Duplex of 3162 is D=2 Bx2)42%(1*6)=24 Duplex of 8137 isD=2 * (@% 7) +2 (1*3)= 1B Duplex of 26734 is =2(2« 4)+2*(6%3)+7? 16+36+49= 101 Duplex of 341628is D=2 «(3 «8) +2 x(4*2) 42% (1x6) = 484 16+ 12-76 The process of finding the square of a number is best illustrated with the help of the following examples. Mlustration 56 Square of $6. Duplex of 4 is D = 42-16, Duplex of 46 is D=2 = (4 = 6) = 48, Duplex of 6 is D = 6° = 36 46? = 36 + 48 x 10+ 16 « 10? Alternatively, 46° = 16/48/34 Mlustration 57 Square of 378. Duplex of 3 is D = 2-9 Duplex of 37 is D=2 (3 » 7) = 42 Duplex of 378 is D = 2 x (3 x 8) + Duplex of 78 is D=2 (7 x 8) = 112 Duplex of 8 is D 116 116 7 aa You have either reached a page that is unavailable for viewing or reached your viewing limit for this book. aa You have either reached a page that is unavailable for viewing or reached your viewing limit for this book. aa You have either reached a page that is unavailable for viewing or reached your viewing limit for this book. 5. 16559225 7. 2101749 6. 588821 8 1752821 Solving Simultaneous Linear Equations “in Two Variables ‘The solution of the system of equations is given by = Dem breh gng y A084 ab, ah, ah = enh provided ay), ~ayb,, #0. For the sake of convenience, this solution is writen in the following pattern aor —__, e-beam yah With the help of the following structure, it is easy to remember the pattem of the above solution. Sd Explanation Below x, we write the coefficient of y and the constant term, Below y, we write the constant term and the coefficient of x. Below I, we write the coefficients of x and y. We put the arrows as shown above, The arrows between two numbers indicate that the two numbers are to be multiplied. The numbers with the downward arrow are multiplied first and from their product subtract the product of the numbers with upward arrows. Mlustration 77. Solve: x+y =7, 5¢+ 12v=7 The given system of equations is xty-7=0 Sx+12y-7=0 By cross-multiplication, we get 9 [xeT=12x-7 7x54 Ix12—5x1 Numbers 1.21 Exercises Solve 1. 2x4 3y=17 3x-2y=6 3.2vty 35-0 4 3x+4y— 65 =0 2v-y-3=0 Sixtyoath ax ~ by Answers Lee, Bx I5y Sxcaye ‘Simultaneous Linear Equation: Mlustration 78 Solve Se-yt fi) Br+2y- Ai) xty-2e «iy Writing equations (i) and (ii), in the form Sr-yt(e~4)=0 Br+2y+(- 52-220 By cross-multiplication, 13 ‘Substituting the values of x and y in equation (iii), we get Bq) +10 B Hence, the solution is x= 1, y= 2andz=1 and y Exercises Solve 1 2e + By + Rx-yt2e=7 v-2yte Bet dy — S25 3-2: 2e-y +3: Baxtyes 4 xt 3y~2 yree3 Bat gy ztxed ye3yt2e=11 aa You have either reached a page that is unavailable for viewing or reached your viewing limit for this book. aa You have either reached a page that is unavailable for viewing or reached your viewing limit for this book. aa You have either reached a page that is unavailable for viewing or reached your viewing limit for this book. 56. 57. $8. 59. 60, a. 2. 8. 64. 65. 66, 4767 exactly divides *** 341, the missing digits are (a) 468 {b) 586 ©) 363 {d) None of these ‘A number when divided by a certain divisor left remainder 241, when twice the number was divided by the same divisor, the remainder was 112. Find the divisor, (@) 370 (b) 365 © 380 (@) 456 ‘Two numbers when divided by a certain divisor give remainders 43 and 37 respectively, when their sum is divided by the same divisor, the remainder is 13. Find the divisor. @ 7 © 7 () 67 @ 7 ‘Two numbers are such that the ratio between them is 3 : 5; but if each is increased by 10, the ratio between them becomes 5: 7, The numbers are @ 3,5 &) 7.9 (©) 13,22 (@) 15,25 Divide 50 into two parts so that the sum of their reciprocals is 1/12. (@) 20,30 (b) 24,26 (©) 28,22 (a) 36,14 The sum of seven numbers is 235. The average of the first three is 23 and that of last three is 42. The fourth number is @ 40 tb). 126 © 0 (a) 195 ‘The sum of squares of two numbers is 68 and the Squares of their difference is 36. The product of the ‘two numbers is (@) 16 (b) 32 (©) 58 (a) 104 ‘What is the least value of K so that the number 6735K. is divisible by 92 @) 5 (b) 7 © 4 @ 3 For what value of K, the number 7236K2 is divisible by 8? @7 () 5 © 4 @ 9 Find the least values of x and y so that the number Sxd23y is divisible by 88. @) 8,2 ) 7.3 © 94 @ 63 24 is divided into two parts such that 7 times the first part added to 5 times the second part makes 146, The first part is @ B ov (b) 1S @ » 67. 68. 70. n. n 2B. 4. 18. 16. 7. 78. Numbers 1.25 Sum of three numbers is 132. First number is twice the second and third number is one-third of the first. Find the secord number. @ 18 () 36 ©) 20 @ 16 What least number must be added to 7231 so that the resulting number is exactly divisible by $ and © together? () 18 (@) 20 (a) 16 © 4 Find a number nearest to 9231 which is exactly divisible by 3 as well as by 1. (a) 9240 (b) 9340 fe) 9540 (@) 9440 Find a nearest number to 12199 which is exactly divisible by the product of the first four prime numbers, (@) 12181 (©) 11281 (o) 12179 @ 129 The sum of squares of tivo numbers is 99 and the squares of theie difference is 46. The product of the two numbers is @ 2 (© 2% (b) 24 @ 2% Ms of that 140% of @ number is 36D, what will be 1 number? 1 b) 204+ ) 20> l a) 2t az, The sum of the digits of a two-digit namber is 8. 1° the digits are reversed the number is increased by 54. Find the number. @ 7 (b) 19 © 21 @) 2 Find the binary equivalent of decimal 117, (a) 1010101 (b) 1110101 (©) INI101 (@) None of these Find the binary equivalent of decimal 52 (a) 110100 {b) 111100 (©) Remainder (d) None of these Find the decimal equivalent of binary 1110101 (@) 110, (b) [hy ©) Ug (4) None of these Find the binary equivalent of decimal 235, (@ loin, (b) 1010111, (©) 11101011, (@) None of these Find the binary equivalent of decimal 701, (@) 1010111101, (b) 1011101101, © ioiii0!, (A) None of these aa You have either reached a page that is unavailable for viewing or reached your viewing limit for this book. aa You have either reached a page that is unavailable for viewing or reached your viewing limit for this book. aa You have either reached a page that is unavailable for viewing or reached your viewing limit for this book. % 10, 12. 13, 14, 15. 16. number and then divided it by 6, geting 112 as the answer. The corect answer should have been (a) 12 (b) 118 1 (a) 124 Which of the following integers is the square of an integer for every integer n? (a) +d (bo) +n © +20 (d) Pent d It and y are negative, then which of the following statements is/are always true? positive IL xy is positive ML x — y is positive (a) Lonly (©) Monly Laty (b) Monty (a) Land III only The sum of the digits of a 3 digit number is subtracted from the number. The resulting number is always (a) Divisible by 6 (b) Not divisible by 6 (©) Divisible by 9 (2) Not divisible by 9 ‘The least number that must be subtracted from cach of the numbers 14, 17, 34 and 42 so that the remainders may be proportional, is (a) 0 (b) 1 © 2 @7 There are four prime numbers written in ascending order. The product of the first three is 385 and that of the last three is 1001. The first mumber is @ 5 (b)7 ou (ir ‘A positive integer, which when added to 1000, gives, a sum which is greater than when it is multiplied bby 1000. This positive integer is @t ©) 5 @7 @3 Of the 120 people in the room. 3/Sthare women. If 23rd of the people are married, then what is the maximum number of women in the room who could be unmarried? (@) 40 (b) 20 (©) 30 (d) 60 Its. . then the value of x3 ~ 6x7 + Gvis @) 3 (b) 2 ot (@) None of these ‘A number of three digits in scale 7 when expressed in scale 9 has its di rmumber is is reversed in order, The TA 19. 20. 2. 23. 24. 25. Numbers 2 1.29 248 148 For every positive real number (by 348 (a) 448 @ © where ({]) is the greatest integer function (a) 0 1 ( [x41] @) fd How many five-digit maltiples of 11 are there if the five digits are 3,4, 5, 6 and 7 in the same order? @ 2 ©) 3B © 1 (a) None of these ‘The smallest number by which 3600 can be divided to make it a perfect cube is @9 (b) 50 (©) 300 @ 450 ‘The least number having four digits which is a perfect square is fa) 1004 (b) 1016 fe) 1036 (@) None of these ‘The remainder when 7 is divided by 342 is (a) 0 ) 1 49 (@) 341 Given m= + x and x is the product of four consecutive integers. Then which of the following is true? (A) mis an odd integer. (B) nis prime. (© nis a perfect square. (a) Both 4 and C are correct (b) Both 4 and B are correct (©) Only 4 is cormect (@) Only C is correet A owo-digit number is such that the product of the digits is 14. When 45 is added 10 the number, then the digits interchange their places. Find the number. @ 2 ) 27 © 9 @ 14 A player holds 13 cards of four suits of which seven are black and six are red, There are twice as miny diamonds as spades and twice as many hearts, as diamonds. How many clubs does he hold? @ 4 (b) 5 © 6 @T In three coloured boxes—red, green and blue—108, balls are placed, There are twice as many in the green and red boxes combined as they are in the aa You have either reached a page that is unavailable for viewing or reached your viewing limit for this book. aa You have either reached a page that is unavailable for viewing or reached your viewing limit for this book. aa You have either reached a page that is unavailable for viewing or reached your viewing limit for this book. 16. cE 78. 1. 81. 82. 83. sum of the third and thrice the first is 90. The third number is @ 2» ow & 2 @% Three consecutive positive even numbers are suck that thrice the first number exceeds double the third by 2, the third number is @ 0 4 © 6 @ 2 If a number 7749584968 is to be divisible by 8 and 9, the respective values of A and B will be (@) 7 and 8 (©) 8 and 0 (© Sand 8 (@) None of these P and Q are two positive integers such that PQ = 64. Which of the following cannot be the value of P4+Q? @ 20 © 16 ©) 6 @% Let <0.50,0 1. Givena set of numbers the middle number, when they are arranged in ascending order, is called the median. So the median of the numbers x, y and 2 would be: (@) less than one (6) between 0 and 1 (©) greater than one (@) cannot say 72 hens cost Rs....96.7 ... Then what does each hen cost, where two digits in place of.” are not visible written in illegible hand-writing? (@ Rs3.23 () Rs Sl (© Rs5.51 @ Rs 722 If m is an integer, how many values of 1 will give - + fan integral value of {162+ 7+ 6) 9 fa) 2 (b) 3 ©4 (@ None of these ‘A certain number when divided by 899 Ieaves the rermainder 63. Find the remainder when the same number is divided by 29. (a) 5 (o) 4 © (@) Cannot be determined The remainder obtained when a prime number greater than 6 is divided by 6 is (a) lor3 ) lors (© 3or5 @) 4ors P,Q and R are three consecutive odd numbers in ascending order. If the value of three times P is 3 less then two times R, find the value of R. f@) 5 b) 7 o> @ 85. 86. 87. 89. 90. on. 92. Numbers 1.33 For the product n(vr+ 1) 2n+ 1), € N, which one of the following is not necessarily true? @ It () Divisible by 3 (©) Divisible by the sum of the squares of first » natural numbers (@ Never divisible by 237 If (BE? = MPB, where B, £, Mand P are distinet integers, then Mf = () 2 (b) 3 o©9 [d) None of these n= 1 +x, where is the product of four consecutive positive integers, then which of the following is/are true? (A) nisodd (B)nisprime (C) nisaperiect square (@ Aand-C only (b) 4 and B only (©) A only {d) None of these The product of all integers from 1 to 100 will have the following number of zeros at the end @ 20 b) 29 © 19 @ 2 the number, found the value of 7th ofthe number It his answer differed from the actual one by 770, find the number. fa) 1584 (>) 2520 (©) 178 td) 1656 If m is any odd number grester than 1, then ni? — 1) is: (a) divisible by 96 always (©) divisible by 48 always (©) divisible by 24 always (@) None of these A,B, C.D, on, Ys Zate the players who participated ina tournament. Everyone played with every other player exactly once. A win scores 2 points, a draw scores | point and a lose scores 0 point. None of the matches ended ina dra. No two players scored the same score. At the end of the tournament, a ranking list is published which is in accordance with the alphabetical order. Then (a) Mwins over N {b) N wins ‘over M (©) M does not play with Hf (2) None of these Two positive integers differ by 4 and sum of their Thea, one of the numbers is, (b) 1 @ 2 ‘ 10 reciprocals is 1° @) 3 © 5 aa You have either reached a page that is unavailable for viewing or reached your viewing limit for this book. aa You have either reached a page that is unavailable for viewing or reached your viewing limit for this book. aa You have either reached a page that is unavailable for viewing or reached your viewing limit for this book. Numbers © 137 Exercise 1.1 1 (a). 2@. 3 (a). 4). 5 (0). 9 (bo). 10 (a). 11 (b). 12 (b). 13 (@). 14 (b). Given Expression = 7372 x (7372 +628) 1372. x 80(0 = 58976000 Let 9999 + 8888 +777 += 19700 x= 19700 ~ 19664 = 36 Lot x 111 = 666666 666666 i Missing figure = 6006 Let 3149 «x= 425115 = A255 = 135 3149 Missing digit = 3 Let the age of Mr. Manoj be (10x + y) yeas. His wife's age = (10y + x) years. Then, (105-7 10y-44) = 1-10) —x or, z+ y= 9x—9y, oF, Bx = 10, y 4 = Sandy =4 “+ any other mmuliple of § make x of two digits] ++ Difference Ox + y ~ 10y — x = Bx 9y = Huy) = 45 ~ 4) = 9 years Let Quotient = Q and remainder = R Then, given 120 = 5R Now, R= 48 = 120 =5 x 483 0 Dividend = 20 x 240 = 48 = 4848 ‘The number when divided by 9, and 13 leaving remainder 6 = (L.CM. of 9. 11, 13)+6= 1293 Required number 294 ~ 1293 =1 Let these parts be x and (24 ~ x), Then, 7x +5 (24-1) = 146 = 13 So the first part is 13 1 atyen sxe 4 64x5 4 xrs64ox 80 15 (d). 16 (c). 47 (a). 18(0). 19 (b). 20 (c). 21 (b). 22 (d). 23 (b). 24(c). Level I dy yel gone 2 2 Let the required fraction be 3 lyse yes Solving these equations, we get Let the numbers be 3x, 4x and Sx Then, Sx+3x = 4x + 52.=9 dy =52 = x= 13 The smallest number = 3x = 3 13 = 39 Let the numbers be a, b, ¢. Then, = Let the numbers be 10x, 15x, 9x, Then, lOx+ 1Sx+9x= 68 => 34r= 689.152 Second number = 15x = 15 x2 =30 Let the required fraction be >. Then, y xt at ax- yt] 2 ee x41 and hay a x-yed z Solving 2x —y~ Hand sy —— 1, we getx=2,y= The fraction is 2 Let the number be x. Then, 61+67+ 71+ 73+ 79=351 Divisor = (24446 - 35) , 79 = 309 Dividend = 456 * 120 + 33 = $8053 Number = 463 x 62 + 60 = 28766 Dividing 43 by 17, the remainder is 9 aa You have either reached a page that is unavailable for viewing or reached your viewing limit for this book. aa You have either reached a page that is unavailable for viewing or reached your viewing limit for this book. aa You have either reached a page that is unavailable for viewing or reached your viewing limit for this book. 80 (a). 81 (c). 82 (b). 83 (2). 84 (c). 85 (a). 10000010011 210 9 aR 9? 76 98 ot 23 gt ot Decimal equivalent m1 x2 Ox 24 Om BH OR? 40x40 x S41 x2 40%2 + 0x2 ep x ohare 2” 2104 22 a! +29 = 1043 toro ob @ bod 2 ts P FF of e Decimal equivatent 18a 12a px Benet rae 2" B+ Ae 42'+ 2 = 59 o101 +1001 i110 11100 +1010 110110 unt 10001 1011 mont 11001 11011 mut io1001t Column 1: 1+1+1=: 2 Queso Remand Column 2: 0+1 +1 +1 (carry from first column) =3) 3 = Quotient 1 and Remainder 1 Column 3; 0-+0++ 1+ 1 (eamy from second column) 2 = 2, ~ Quotient | and Remainder 0 Column 4: 1 +1 +1 (carry from column 3) 4 4,5 = Quotient 2 and Remainder 0 Colurnn 5: 1 +1 +1 +2 (carry from cotumn 4) 5, Sig = 101, ‘Note: Quotient in any column is carry for next column, 86 (b). 87 (). 88 (a). 89 (b). 90 (a). 94 (c). Numbers © LAL 1000 101 mu 100100 Mm Jol on Tt 111000, M1001, = 16 = 8+ I Since 11001, < 111000,, so we are to subtract a lower number fiom a higher number. Making the digits equal in the number to be subtracted, we get 011001 ‘Complement of 01 1001 = 100110. Adding 100110 to 111 000, we get 111000 100110 iornniio [1 in the [ ] is the 1 carried over} Adding | tothe extreme right digit in O11 110, we get ono endl Wut 111000, ~ 11001,= 11 10001, = 24 +1 = 17, Un,= 842242! +1215 Since 1111, < 100015, we are to subtract a lower number from « higher number. aa You have either reached a page that is unavailable for viewing or reached your viewing limit for this book. aa You have either reached a page that is unavailable for viewing or reached your viewing limit for this book. aa You have either reached a page that is unavailable for viewing or reached your viewing limit for this book. ‘The following chan shows the number of ways in which it ean be formed. Number of ways in Digit at ten’s Number of ways in which unit's: place, place, Le, y which unit's place x tan be filled ie.” cam be filled 1 (ie) 2 2 Fie. 0, I) 2 fie, 1,2) 3 3 (ie. 0 1,2) B Gee 1.2.3, a08) 9 9 (12, 0,1,2,.9) Total number of possible 3-digit numbers = (1x2) 4(2*3) +B x4) HEX S) Hot (Te 8) + (8 9) = 240 3 (b). Product (n~ 1) (n—2}... 3 * 2% 1 is not divisible by # if wis 4 ora prime number. We have to find the number of primes in 12s ns40, + 413,17, 19,23, 29,31,37} Number of positive integers in the range 12S s40is7 4/(d). Take any arbitrary value of x and y Let 2 (prime number) 50 (composite number) Going through the options, (a), (b) and (c) are wrong because y — x, xv and xty és are even integers for x = 2 and y None of the staiements are true. 5 (a). The set of prime numbers 3, 5, 7 is the only set which satisfies the given condition. 6 (d). If 4? is divided by 6, remainder is 4 If # is divided by 6, remainder is 4 If 44 is divided by 6, remainder is 4. vided by 6, remainder is 4, Method (mod 6) = (49)! (mod 6) = (444 (mod 6) remainder = 4 4 (mod 6) 7 (a). 8(d). 10(c). 11 (c). 14a). 15(d). 16 (a). 47 (a). 18 (a). Numbers | 145 Let x be the number, (r+ 1926= 112 5 242 22 x= 126-12 x= 67-12 = 660 Correct answer = 2412 6 60 - Bei2= 10+ 12-12 (ne Pst dn ed Let the three-digit number be 10D. + 10y + = (10x + 10y +2) Hy +2) = 99" +9 =9(1Le +9) which is always divisible by 9. Let x must be subtracted from 14, 17, 24 and 42 such that (sa? = (7 =) G4~ x) =ae=2 ‘Number of women inthe room = 2120 = 72 Number of married people = =»120 = 80 Number of unmarried people = 40 Number of men in the room = 48 Ill the men are supposed to be married, then ‘number of married women could be 80 48 ~ 32. Meximum namber of unmarried women could be 72 40 x=24+254213bx>2- For x= 2,93 - 6 + 6r= 4 Fors = 3,28 62 + G9 62 +6 <0 248 in the scale of 7 9 itis written as 305 written as $03, In scale Given expression = 53647 isa multiple of 11, because the difference Of the sum of the digits in the odd places and the sum of the digits in the even places is divisible by 11, because G6+6*7)-G+4y= Ul ‘Total number of five-digit multiples of 11 = 3! (Permutation ofS, 6 and 7 in the odd places) * 21 (Permutation of 3 and 4 in the even places) =6x2= 12 aa You have either reached a page that is unavailable for viewing or reached your viewing limit for this book. aa You have either reached a page that is unavailable for viewing or reached your viewing limit for this book. aa You have either reached a page that is unavailable for viewing or reached your viewing limit for this book. 82 (a! 83 (b). 84 (c). 85(d). 86 (b). 87 (a). 88 (b). Since m is an integer, hence for the entire expiession to become’ an imeger 2) stout be an integer. And (© can be an integer for n= 1,2,3, 6. Hence n Will have four values. Dividend — Divisor » Quotient + Remainder = 3999+ 63 Dividend = 29 «319 +29 x2+5 = 2919+ 2)+5 Let us solve the question for some prime numbers greater than 6 ie., 7, 11, 13 and 17. If these numbers are divided by 6, the remainder is always cither 1 or § P, Q and R are three consecutive odd numbers Therefore, Q= P +2 and R= P~4 Given: 3P =2(P + 4)-3 =P Hence R 8. Its clear that for n = 237, the expression n(n + 1) (Qn I)is divisible by 237. Hence, option (d) isnot necessarily true. Given (BE)? = MPB. BE is a two digit number and will be less than or equal to 31 because (32 is a four digit number, B can tate any of the three values, 0, 1, 2 and 3. But it eannot be 2 and 3 because none of the square produces the digit 2 and 3 at the unit place and it is the requirement because B (the first digit of BE) should be equal to B (the last digit of MPB). I is hence confirmed that B should be either 0 or 1. But it ean not be 0 because square of none of two digit number starting with 0 produces three digit number ending number whose square ends up wing 1 and such number is 9. Hence, 3E = 19. Therefore (19)? = 361. Hence, M= 3 Let us solve the question for first four consecutive positive integers, then x= 1 = 2x = 3*4~24, 2 m= 1+ 24 = 25, we find that » is odd and 1 perfect square. This is true for any set of four consecutive positive integers Number of zeros in the product of any number is decided by the number of 2 and 5, whichever is less. Every combination of 5 and 2 will give fone zero. In the product ofall the numbers from 1 to 100, the number of 5 will be less than that of 2. Hence, the problem will be solved if we know the number of $ in the product of (1 to 100). Clearly, there are 20 members which are divisible by 5. Besides, there are four numbers 25, $0, 75 and 100 which will have one addition 5. Hence total number 5 will be 20 + 4 = 24. Therefore, number of zeroes in the product of all the numbers from 1 10 100 is 24 s+ 89 (a). 90 (c). 94 (a). 92 (a). 93(b). 94 (0). 95 (a). 96 (a). Numbers 8 149 56 17 Given: Sx- e705 700 18 144 = xo 1584 Lets solve the question for any two odd numbers greater than 1 ie., 3 and $ then nf’ — 1) for m= 3 is3 «8 = 24 nor ~ 1) for n= 5 is S * 24= 120 Using options, we find that both the numbers are divisible by 24 It is given in the question that ranking is in accordance with the alphabetical order. It means, 4 occupies first, B second, C third, D fourth position and so on. In other words 4 wins all the matches, B wins all the matches except with 4, allthe matches except with 4 and B and In view ofthe above order N winsall the matches except with 4 to M. Hence Mf wins over WV. Let one number be x, then second number will be w +4) L dis x Ged xert4 10 2e+4 _ 10 xGed 2 Mees) OT = x83 Unitaligit in (2)*=6, = 6, (2)!°=6. Hence 2hasa cyelicty of four. Hence unit digit in(2)"*=6 Therefore, unit digit in (2)5! = (2)** < (2) - 6x88 If mis odd then and 1? will also be odd. It can be checked for any odd integer. fm m=27 n=0u=t 2 = 3 n= 23-7 y= 15 n=4,ug=31 From the pottern, it is clear that Hence, iyo = (2)— 1 = 1023 By the rule of divisibility for 8, any number is visible by 8. if the number formed by the last three digits is divisible by 8. And the same rule ‘will be applicable to find the remainder. Now the last three digits in the hundred digit number of the form 1234567891011 121314... i 545, Therefore the remainder when S45 is divided by 8 is 1 aa You have either reached a page that is unavailable for viewing or reached your viewing limit for this book. aa You have either reached a page that is unavailable for viewing or reached your viewing limit for this book. aa You have either reached a page that is unavailable for viewing or reached your viewing limit for this book. Exercise 1.5 1 3 2 Mahesh was able to find the two natural numbers both greater than one as soon as their product was {oid to him and no other information was given, The sum ofthe nwo numbers is 16. What wes their product, ifitis greater than 402 a) 48 (b) 55 (©) 60 (a 63 ‘The symbols xx, )2 denotea pair of 2-digit numbers in which 0s not permitted inthe left most place. Similarly “1001 denotes a 4-digit number with middle digits zero, The number of solutions ofthe equation (x¥){V2)= 100 (a) 16 (by 15 ©) 9 @7 (a) denotes the number of positive divisors of including | and itselt), where ‘nt! isapositive int less then 10, If cr? )~3 then how many values of 7 are possible? @ 9 b) 8 © 5 @ 4 The positive integers p and q are such that 1p isdivided by 4 1 5 Ina certain base system, a number is written as 3326. Which of the following cannot be the value of the aiven number in desimal sysiom? ) 3 (d) None of these (a) 74 (b) 1203 fo 158 (a) 2463 n= 2 y', What is the total number of n within the first 3000 natural numbers? (where x and y are natural numbers) a) 0 (b) 37 19 (a) None of these How many numbers of the form n~ 7k Sp but notof the form 2m, where &, and mate positive integers, cexist in the first 1000 natural numbers? @ ) 4 () 8 {d) None of these When ((32)")" is vided by 7, the remainder @ I {b) 2 4 a 5 Arnumberay dye ay in base 9 representation has the value a, 9" a1 9" 1+. + a9 +ay, For example 372 inbase 9 has the value 3 81+7% 9-2, Whatis the units digit of the number (1002)! where all the numbers are expressed in base 9 representation (2 ) 8 4 7 Five consecutive prime numbers are such that the sum of the second and third is less than the fifth by one, If the prime numbers lie before the 10th prime number, then the five primes under consideration will add up to (@ 30 (b) 39 2 (@) 38 aa You have either reached a page that is unavailable for viewing or reached your viewing limit for this book. aa You have either reached a page that is unavailable for viewing or reached your viewing limit for this book. aa You have either reached a page that is unavailable for viewing or reached your viewing limit for this book. Es Genres ed The least common multiple of two o more given numbers is the least or lowest number which is exacily divisible by each of them. For example, consider the two numbers 12 and 18, H.GE, and LOM, of Numbers) 23 Multiples of 12 are 12, 24,36, 48, 72, Muttiples of 18 are 18, 36, 54, 72, . Common multiples are 36, 72, Least common multiple, ie. LCM. of 12 and 18 in 36 METHODS OF FINDING L.C.M. ‘A. Method of Prime Factors Step 1 Resolve each given number into prime factors, Step 2 Take out all factors with highest powers that occur in given numbers. Step 3 Find the product of these factors. This product will be the LCM. Mlustration & Find the L.C.M. of 32, 48, 60 and 320, B=21 Solution R20 = 266 LOM.=2«3« $= 960 B, Method of Division Step 1 separated by common, Step 2 Divide by any one of the prime numbers 2,3,5, 7, 11. Which will divide at leastany two of the given numbers exactly, The quotients and the undivided numbers are written ina fine below the first Step 3 Step 2 is repeated until a line of numbers (prime to each other) appears. Step 4 Find the product of all divisors and numbers, in the last line which is the required L.C.M The given numbers are written in a line Illustration 9 © Find the L.C.M. of 12, 15, 20 and 54 Solution 15, 20,34 [6 15, 10, 37 3/315, 27 spa 5, 9 “Thok 9 LOM. =2*2*3«S"1 [x1 * CAUTION Before finding the L.C.M. or H.C.F,, we must ensure that all quantities are expressed in the same unit, ME USEFUL SHORT-CUT METHO! SC 1. H.CE. and LCM. of Decimals Step 1 Make the same number of decimal places in all the given numbers by suffixing zero(s) ifnecessary. Step 2 Find the H.C.P/L.C.M. of these numbers without decimal Step 3 Putthe decimal point (in the H.C.E/LC.M. of step 2) leaving as many digits on its right as there are in each of the numbers, llustration 10 Find the L.C.M, of 1.2, 0.24 and 6, Solution The given numbers can be written as 0.24 and 6.00. Now, ignoring the decimal we find the L.CM. of 120, 24 and 600. LOM, =242%2%3 «5% 1%5 = 600 Thus, the required LCM. = 6.00, ie 6 Find the H.CF, of 616 and 13 Solution The given numbers can be written as 6.16 and 13.00 Now, ignoring the decimals we find the H.C.F. of 616 and 1300. 16) 1300(2 Wlustration 11 H.C. of 616 and 1300 is4 Thus, the required H.C.F. = 0.04 aa You have either reached a page that is unavailable for viewing or reached your viewing limit for this book. aa You have either reached a page that is unavailable for viewing or reached your viewing limit for this book. aa You have either reached a page that is unavailable for viewing or reached your viewing limit for this book. 20, 21. 2, 24, 28, 26. 27. 28, (a) $4 (b) 64 7 (a) si Find the greatest number which will divide 3962 4085 and 4167 leaving the same remainder in each case, (a) 37 (b) 39 (41 @) 43 A wholesale tea dealer has 408 kilograms, 468 kilograms and 516 kilograms of three different qualities of tea. He wants it all to be packed into boxes of equal size without mixing. Find the cepacity of the largest possible box, (a) 50 (b) 36 (o 24 (9) 12 A room is 4 metres 37m long and 3 metres 23 m. broad. It is required to pave the floor with minimum. square slabs. Find the number of slabs requited for this purpose? (a) 485 (b) 431 (© 391 (a) 381 ‘The least perfect square number which is divisible by 3, 4, 5, 6 and 8 is (a) 900 (b) 1200 (©) 2500 (a) 3600 Find the least number of five digits which when divided by 12, 16, 21, 36 and 40 leaves in each case a remainder 8. (a) 10088 (b) 10072 (©) 10080 (d) None of these Three pieces of timber 42 m, 49 m and 63 m long have to be divided into planks of the same length. ‘What is the greatest possible length of each plank? @ 7m () 14m (9 42m (d) 63m ‘Vhree men stert together to travel the same way around a circular track of 11 kilometres in L circumference, Their speeds are 4 $ and 8 kaw, respectively. When will they meet at the siarting point? (@) 11 hours (b) 12 hours (c) 23 hours (d) 22 hours Five bells begin to toll together and toll at intervals, 0136, 45, 72, 81 and 108 seconds. Afler what interval of time will they keep on tolliag together? (a) 3240 seconds (b) 3080 seconds (©) 3140 seconds (d)_ 3200 seconds ‘Three different containers contain different quantities, of mixture of milk and water, whose meesurements are 403 kg, 434 kg and 465 kg, What biggest measure ‘must be there to measure all the different quantities exactly? 29, 30, a 32, 33. 34, 36. 37. 38, HCE. and LCM. of Numbers !! 27 @ Tks (bi 7ke (©) 31kg (a) 4tke The L.C.M, and G.C.D, of two numbers are 1530 and SI, respectively. Find how many such pairs are possible? (a2 () 3 4 (d) only one Find the least number of five digits which when divided by 63, 56 and 42 leaves a remainder 1 in cach case. (a) 10082 (b) 10081 (©) 10001 (a) 10071 The H.CF. and L.C.M, of two numbers are 44 and 264, respectively. If the first number is divided by the quotient is 44. The other number is, @ 3 (b) 66 (©) 1B2 (a) 264 The largest natural number which exactly divides the product of any four consecutive natural numbers, is (a) 6 () 12 24 @ 120 Find the least number of six digits which divisible by 15, 21 and 28 (a) 100480 (b) 100270 (©) 100380 (@) 100340 Find the greatest number of five digits which when exacily divided by 12, 15, 21, 25 and 28 leaves 5, 8, 14, 18 and 21 as remainders, respectively. (a) 98696 (b) 98700 (©) 97695 (0) 98693 What is the smallest number which when inereased by 3 is divisible by 16, 24, 30 and 327 fa) 480 (b) 475 te) 472 (@ 477 ‘The least number of square tiles required to pave the ceiling of a room 15m [7 em long and 9m, 2em broad, is (a) 656 fe) 84 (b) 738 (@) 902, Find the least number which when divided by 2, 3, 4, 5 and 6 leaves 1, 2, 3, 4 and 5 as remainders, respectively, but when divided by 7 leaves no remainder, fa) 210 () 126 (b) 119 (d) 154 Find the greatest number of five digits which when divided by 4, 6, 10 and 15 leaves the same remainder 3 in each case. fa) 99993 (©) 90003 (b) 99063 (a) 99063 aa You have either reached a page that is unavailable for viewing or reached your viewing limit for this book. aa You have either reached a page that is unavailable for viewing or reached your viewing limit for this book. aa You have either reached a page that is unavailable for viewing or reached your viewing limit for this book. HF. and LOM. of Numbers 211 Exercise 2.2 Level I L@ 2% 2@ 40 FM 6) o. &w& 20 mo mH Re BH He Exercise » Quéstions ftom Compeiitive Examinations Le 200 20 40 &@ 60 FH. 8 28 2 nO RH BH Me BM we no Re ww ___ HINTS AND SOLUTIONS, j Exercise 2.1 Level I 1 (c). H.C.F of 27, 18 and 36 10.20 5 (0). WCF of 20 and 2 is)27(1 H.C, of 27 and 181s 9 HCE of 94nd 36 9 Now, 36 H.C.E, of 9 and 36 is 9 Tieriiesnemsi H.C.F of 27, 18and 36189, 6 2 (a). LOM. of Z, 5 and _ LCM. of 2, 3 and 6 H.C. of 5, (0 and 25 LCM. of 2, 3 and 6 = 6 and, HCE. of 5, 10 and 25 = 5 Required LCM. = & 3b). 2 | 25, 30, 35, 40 Required LCM, = 2» 5 = 4200 H.C, of 852 and 1065 is 213 HCE. of 213 and 1491 is 213 <7 4a). LOM. of 8.21 nd @ H.C.F. of 4, 10 and 20 = 2 and L.C.M. of 9, 21 and 63.= 63, 2 + Required HCE = 5 L.C.M, of 16, 18, 20 and 25 is 3600 Required number ~ 3600 « K+ 4 Tx 51442)K +4 = (7% SIA) K 42K 4 Now (2K + 4) is divisible by 7 for K = 5 Required number = 5 * 3600 ~4 18004 6 (c). 7 (c). H.C. of 165, 195 and 85 will be maximum area of cach of the flower beds. H.C. of 165 and 195, 165 9501 165 TOS{ 2 150 H.CF. of 165 and 195 is 15. Also, now, H.C.F. of 15 and 85 is S aa You have either reached a page that is unavailable for viewing or reached your viewing limit for this book. aa You have either reached a page that is unavailable for viewing or reached your viewing limit for this book. aa You have either reached a page that is unavailable for viewing or reached your viewing limit for this book. 8 (bd). 43k +431= 430 => 43+ = 430 = k+l=10 But &, / must be co-primes. So (k= (1, 9and(3.7) 9(c). Required length=H.C.F.of (42, 49, 63)=7m 10 (b). 56 = 1 * 56 = HCE is 1, hence number of files = 56 228=9FLCR.is2,hence numberof tiles +14=15 4014 = H.C Fis2,thereforenumber of tiles =247=9 7 8=9 H.CE is |, therefore number oftiles =7+8=15 Hence, the minimum number of tiles ean be 9 and thedimension ofa tik is 2 metre each side. Thus(b) is correct option HR of 04,82, 162,20)=24 LCM, of 24, 8%, 167, 20°)= 28 x 125 = 32000 14 (b). Exercise 2.3 H.GE.XLCM. rst number _ 480% 160 © 80 3 (b). LCM, of 60 and 62 secondsis 1860 ssc =31 min 4 (d). Second number = 600 They will beep together at 10.31 a.m, 4 (b). LCM. of 6, 7, 8, 9 is 504. Bells will ring together after an interval of every 504 seconds. Number of times that the bell will ring in 2 hours, i.e., 7200 seconds 100 i, ta times. 50h 7 Since bells rang together in the start also, therefore the bells will ring 15 times in 2 hours. LCM. of 48, 72, 108 =432 2+ Light will change simultaneously after every 432 seconds, ic, after every 7 min and 12 seconds 5 (e). Let the number be x and 4r then 84. 21 =x * 4x 6(c). 764 41 => x=21 84 7(d). LCM. * H.C. = produet of numbers 480% 160 am 7 1600 Other mumber = 12(a). 43 (b) 14 (0). 8 (0). 9(@). 10 (a). 11 (b), 12 (b). 13 (b). HCE. and LOM, of Numbers #215 Prime factors of 35= 57 85=5%17 LOM, 35 85 Sa least possible value of ST 1Te 13 4+2-86 LOM. of (7, 12, 16) =336 If we divided 1856 by 336 then remainder is 176. Since it is given that remainder in this condition is 4. Hence, the least number to be subtracted = (176 ~ 4)= 172 Required aumber= L.C.M. of (8, 11, 24) = (264 - 5) = 289 The bells will chime together again after a time that is equal to the L.C.M. of 18, 24 and 32 = 288 min ~ 4h and 48 min, Required number is calculated by the L.C.M. of Q, 3,4, 5, 6) ~ (common difference) In this case, common difference = (2-1) B-2)-(4-3)-(5-4)-(-9)-1 2. Allinieger of the set willbe given by (60n~ 1) If n= 1, (6001) = 59 If n= 2, (60n—1) = 119 Since range of the set A is between 0 and 100, hence there will exist only one number ie., 59 aa You have either reached a page that is unavailable for viewing or reached your viewing limit for this book. aa You have either reached a page that is unavailable for viewing or reached your viewing limit for this book. aa You have either reached a page that is unavailable for viewing or reached your viewing limit for this book. ‘A number multiplied by itself is known as the square of the given number. For example, square of 6 is 6 6= 35. Square Root Square root of a given number is that number which when multiplied by itself is equal to the given number. For example, square root of 81 is 9 because 9° = 9x 9=81 ‘The square root of a number is derioted by the symbol ¥ or J”, called the radical sign. Thus, J81 = 9, Y64 = 8 and so on. Note that Vi = 1 Methods of Finding the Square Root 1. Prime Factorization Method 1. Find the prime factors of the given number. 2 Group the factors in pairs. 3, Take one number from each pair of factors and ten muliply them together. ‘This product is the square root of the given number. Mlustration 1 Find the square root of 4761 20705 Solution (i) 4761 = 23*23 x 3x3 : 2 VART = 23 «3 = 69, (ii) 207025 = $*5 x 7x7 * 1313 AWB = 5 «7x 13 = 458, REVIEW OF FUNDAMENTAL CONCEPTS S826 IMPORTANT ‘The above method is used when the given number is 1 perfect square or when every prime factor of that umber is repeated twice. I. Method of Division This method is used when the number is large and the factors cannot be easily determined. The working rule is explained with the help of following example: Step 1: The digits of a number, whose square root s required, are separated into periods of two beginning from the right The lat period may be either single digit or a pair. 416 8 81] 2EB 6 6s 609 946] 5676 5676 Step 2: Find a number (here, 4) whose square may be equal to or ess than the first period (hete, 22) Step 3: Find out the remainder (here, 6) and bring, down the next period (here, 65) Step 4: Double the quotient (here, 4) and write to the left (here, 8). ‘The divisor of this stage will be equal to the above sum (here, 8) with the quotient of this stage (here, 7) suffixed to it (here, 87). Repeat this process (step 4 and step 5) tll all the periods get exhausted. ‘The quotient (here, 476) is equal to the square root ofthe giveri number there, 226576). Step 5: Step 6: aa You have either reached a page that is unavailable for viewing or reached your viewing limit for this book. aa You have either reached a page that is unavailable for viewing or reached your viewing limit for this book. aa You have either reached a page that is unavailable for viewing or reached your viewing limit for this book. 1 1 @ FR sv3 @1 oe 24, 2484524144 = 1 @ 14 (b) 16 (©) 166 (@) 188 25. 0.0009 + VOOT =? @3 (©) 0.3, oy (d) None of these % DR 2 1 1 30-43 © se (e) 3-297 @) 3422 54 Jor then x is equal to (oy 324 (@ 4800 4 (@) Greater than 6 (be) 2 @ 2 T6F 30. If Yi2 = 3.404, value of 7 + 2 is @ 307 (b) 3.464 (© 31753 @) None of these 31. If JT5G25 ~ 125, then the value of V15625 + 136.25 + ESE is (@) 13875 (b) 13.875 . (© 13875 (@) 15625 If YOORROIKG = 0.03 * 03 x J, value of Z is (@) 0.009 () 0.03 © 0” (@) None of these Given that /4096 = 64, the value of ,/4096 + {0.96 * ¥.004096 is @ 704 (&) 70.464 © T1104 @ 14 tf i+ f1-22 2 142, value of x is 2401 7 33. 34, 36. 38. 39. 40. 4. 2. 4B. 44. 45. Square Root and Cube Root | 38 (a) 3 (bo) 1 os @7 Which of the following numbers, wherein some of the digits have been suppressed by symbols, can possibly be the perfect square of a 3 digit odd number? () 68 1 @) oe (©) 1x4 (4) 9 xx THREE, Vi 36250. 0289%72.9%64 (a) 24 (b) 2.40 (e) 0024 (2) None of these Find the eube root of =. ind the cube root of 3, fa 10s (b) 1625 (wis () None of these TOT TRIO =? (03 &) 003 © Sore (4) None of these Find the cube root of 15.625 (a) 35 (b) 25 (43 (a) 33 What is the value of ¥° 000068? a) 04 (&) 008 © 004 (@) 016 What is the value of {Jaa Tee Va? @ 3 ) 5 (97 (9 The least number by which 14175 be divided to make it a perfect square is? (a) 3 (b) 5 (oy 7 (d) 15 N5-N3 “Bahr it equal 0 fay 4+ 15 (b) 4 ~ VIS {e) + «1 oR + VIG 96 (@) 26 (b) 6yF 2 @2 OR Given that J) = 4. 472, find the square root of 2 25; up to two places of decimals. @ 136 (by 169 © 149 (d) None of these aa You have either reached a page that is unavailable for viewing or reached your viewing limit for this book. aa You have either reached a page that is unavailable for viewing or reached your viewing limit for this book. aa You have either reached a page that is unavailable for viewing or reached your viewing limit for this book. 14 143-J2) 4G 14 (a). 4 = G89) BGC) ©. 38” G-G+iD) 9-2 23 - v2) = 23 - 1.414) 2 1.586 = 3.172 15((c). The number nearest to 3579 which is a perfect square is 3600, Required number = 60? ~ 3579 = 21 ‘ FT x . f+ ZZ) wee tea [2t) 145 17 (0). 18 (a). J0.016a = 0.016 » Vi = Vi3F% = yuo MOH _ x 20 (d). Let 5 wos Th, y 225 225 or, 36 . = 6x15 * 100 10 21 (b). Vi76+J3401 = JI76Fa9 = JOS = 15 22 (a). Vi0xV15 = ¥150 = J25x6 = VBxV6 = 5/6 A li BO EA wi a8 24 (b). 248+ 524 Jiaa = \248+VS2412 = Bae V248+8 P56 = 16 Square Root and Cube Root 39 25 (b). 26 (A). 27 (b). 28 (b). Let, given expression = x Then, px = 3912 +x = Yo x- 1220 or, (e-4)(E+3)=0 So, x= 4 (neglecting x = -3) 2 29 (qd). Given expression 12, 2 x 30 (c). 31 (c). Given expression T36a5 + [625. 100 12S, 125 (iseas 10000 10 100 125 + 125 + 1.25 = 138.75 32 (a). J0.03x0.3xa = 0.03 x 03 x Vb - & 2s. TEXOS = 7003 «0.3 a *, 7 = 0.009 33 (b). 14096 + (90.96 + Y004056 4096 , [4096 = A096 + JAE, |_A08 *Vt00 *¥ T000000 oe [000 34 (b). 1 aa You have either reached a page that is unavailable for viewing or reached your viewing limit for this book. aa You have either reached a page that is unavailable for viewing or reached your viewing limit for this book. aa You have either reached a page that is unavailable for viewing or reached your viewing limit for this book. Teas Itis a common need to simplify the expressions formulated according to the statements of the problems relating to practical life. To do this, it is essential to follow in sequence the mathematical operations given by the term BODMAS. BODMAS Each letter of the word BODMAS stands as follows B for Bracket toy ‘There are four brackets, namely, — bar, (), { } and [ J. They are removed, strictly in the order — ©, 4} and [] O for OF 2 of D for Division -M for Multiplication A for Addition : 5 for Subtraction - ‘The order of various operations in exercises involving brackets and fractions must he performed stricily according to the order of the letters of the word BODMAS. +x Note: Here, 5=§ = ~ (3) =3 IMustration 1 Simplify stfategt opstatui-/3 2 [S02 8 Solution Given expression : [ited oettefu-(o-F 3) = #is.2 ite... PLS 2 a wi 2 MG 8 2s ae 9 2 [ [ 16 8] x | ope 2 [5°24 8} 2 120° 8 17_f16+1035] _ 17_ 1051 2 L 120 2 120 1020-1051 31 120 Mlustration 2 Simplify “st [yt Hadghl fe Solution Given expression Use of Algebraic Formulae ‘The following important formulae are sometimes found useful in dealing with the simplifications, aa You have either reached a page that is unavailable for viewing or reached your viewing limit for this book. aa You have either reached a page that is unavailable for viewing or reached your viewing limit for this book. aa You have either reached a page that is unavailable for viewing or reached your viewing limit for this book. of the lowest term ‘Multiply the numerator and denominator 33 z 28 C vee 4 160 Multiply ts ‘numerator and ai of the’ Lx by 33 toget 33 (33) 160 tem 33 480-33 447 _ 4127 160 ~ 160 ~ “160 2. Comparison of Fractions The following points ar¢ found useful while comparing two or more fractions. (@ Ihe denominators of the fractions are same, the ne whote numerator is the largest. lustration 16 Which is the largest fraction among (©) If the numerators of the fractions are same, the largest is one whose denominator is the smallest. llustration 17. Which is the largest fraction among (© Hneither the numerstors nor denominator: of the fractions are same then they are converted into equivalent fractions of the same denominator by taking the L.C.M. of the denominators of the given fractions. Then, the fractions are compared according to ( Mlustration 18 Which the largest fraction among 55 Sandy Solutio LCM. of 2, 3,5 and 8 = 120 1 _ 1x60 _ 60 Then, 2 2x60 ~ 120 2 _ 2x4) _ 80, 3 ~ 3x0 ~ 120 LA 4x24 96, 5” 5x2 120 3_5xis_ 75 me a” Bas ~ 120 Sinplficaton v4.5 Now, the denominator of these fractions are same and the largest numerator is 96. Hence, the largest fraction, 6 5 Which is greater © or 59 Mustration 19 Which is great B 7 Solution Step 1. Cross multiply the wo given fractions bys nAz we get 6 «7 =42 and 13 x5 =65 Step 2. Since 63 is greater than 42 and in 65, the 5 5 amerator of > is included, . © is greater wn of s is incl 7 ae than B (If the difference of the numerator and denominator of esch of the given fractions be same then the fraction of the largest numerator is the smallest. istration 20 Which one of the following fractions is the largest? Solution Since in each of the given fractions the difference between the numerator and denominator is same and the largest numerator is 9, therefore, the largest fraction is > 10 (© nthe given fractions, =, 224, yp yab” y+2b S270 where a 4 4 1 oF Os Ife * y= +2Py- 2) then 7* 5 =? @ mH (b) 243 © @) If'm and m are whole numbers such that m" = 121, then (m— 1)"1 =? @ 10 © 0 &) 10 @ 10 Between two fractions 3 aad i. ‘how many fisetions are there in all? @ Four (©) Sixteen (b) Zero @ Infinite si. 82. Simplification 1 49 A boy was asked to maltiply a given number by and got the result 225 more than what he should have got if he had multiplied the number by 8/17. The given number was @ 8 on (c) 64 (@) 136 The value of +t is evame ot 2 23°34" 9.10 ul 8 ig 5 9 2s 6 © 089 55. 57. 58. (a) 6 (b) 3 9 (a) 12 If we multiply a fraction by itself and divide the product by its reciprocal, the fraction thus obiained 26 is 1857. The original fraction is by 22 0) 25 () None of these "7 a+b = 1, att is 222 quad 0? HZ what is 2 equal n 7 @ a 0) = 2 2 ot CO A tin of oil was Son full, when six bottles of oil were taken out and four bottles of oil were poured into it, it was i full. How many bottles of cil can it contain? (a) 10 (b) 20 © 30 (d) 40 In an examination a student was asked to find 3 wy oracent is umber, By mistake he found 3h fit, His answer was 150 more than the correct answer The given number is @ 1% ) 240 © (8) 250 The value of -s2-32-5} (2-i5) is aa You have either reached a page that is unavailable for viewing or reached your viewing limit for this book. aa You have either reached a page that is unavailable for viewing or reached your viewing limit for this book. aa You have either reached a page that is unavailable for viewing or reached your viewing limit for this book. 32. 33. 34, 35, 36. 31. 38. 39. @ Ww ow 3 © We (@) None of these ‘The largest number in the sequence 1,27,3* 44 is \ @1 27 OR wo 4 5 “io 8 z of 24 is equal to a of what? @ 5 (b) 8 7 os @7 If p is @ positive fraction less than 1, then (@) 2~p isa positive number 2 (0) # is less than p L 4 © + isa positive integer > (8) + is teas than 1 ? More than half of the members of a club are ladies. If a 2 & of the ladies and 7 of the gens in the club attended the meeting, then what isthe smallest number cof members that tho club could have? (@) 25 (b) 18 Ox» @ 2 The term (77 + 5)" is approximately equal to r+. Which of the following is the closest approximation to (85)? (2) 9.06 (b) 9.34 © 922 (@ 928 2 « 3° is divisible by @ 2x3? (by 26 « 38 (@) 2x3" @ 2x3 2g What is the value of =2—2— 2 DT: a*9"9 z 7 ©F OF 49 7 oF OF 885*885* 885+ 1151156115 85% 8854 115% 115 — 885% 115 (a) 1000 b) 7 © 85 @ 15 40, 4. Ce 4. 44, 48. 46, a. 48. Simplification 4.13 (a) 927 (4s (©) 364 (@) 322 The value of — 23)" —0027__ 5 By? +0.69+009 @2 © 3 (@) 2327 @ 2273 The value of (eS) «(HB )t)s ee OTe saa) aa 1 1 OTs Os 1 +3 Orsay on The expression 2 (X= 2)(42 95+ 19) empties to (= G? =3x42) @ @-) ® @-2 © @-7) @ We-7) The value of lee lee Tre x0-) wt ® +x @ 1-7 @l If aby"! = (b/ay*, then x is equal to @ 1 @) 12 © 72 @2 The expression +e 1 TeaPay ee exo xe fo) he ®1 © 0 (@) None of these 1 Tra * Te gme eal fa) 0 ) 1 © 12 @® a Consider the following statements W) Ifa" = 6, B= 6. =a, then xyz = 0 (B) If p = a',q =a", (pg = a", thenxyz= 1 or ab + be + ca = 0, then xyz = 1 OF these statements aa You have either reached a page that is unavailable for viewing or reached your viewing limit for this book. aa You have either reached a page that is unavailable for viewing or reached your viewing limit for this book. aa You have either reached a page that is unavailable for viewing or reached your viewing limit for this book. 6 (c). 8 (a). 9 (c). or xs 3 We know that 4° =>" _ @ +ab+b? The given expression = 0.8 Given expression 1-R-t-@-Di -R-8-3) (2-2 Given expression Simplification 84.17 B 7 =3+|3+. =3+[3 3+)3 foxtit| 3 [xg] 10 (a). Given expression _ (69842)? ~(30158)? 69842-30158 _ (69842 8)(69842 + 30158) (69842-30158 = 100000 14 (d). Given expression 12 (b). Given expression 79-225 = 0.50 13 (c). Given expression txtean 4 gs Nl 2420 °«8 23 14 (c). Given expression _ (0.53)? -2x0.53x0.41+(0.41)* a2 nF _ (12)? | ; “on 70 15 (). Given expression = G2? x(3x3x2)4 - 7316 34 x38 x24 3 16 (¢). Given expression =i =1si+ 2+ 243 aa You have either reached a page that is unavailable for viewing or reached your viewing limit for this book. aa You have either reached a page that is unavailable for viewing or reached your viewing limit for this book. aa You have either reached a page that is unavailable for viewing or reached your viewing limit for this book. ath etd ab ond 65 (a). Putting x for ? and solving (1.06 + 0.04)? - x = 4 « 1.06 * 0.04 Here, 1.06 = a and 0.04 = b (at bf x= dab f= (at bf ~ fab = (a ~ 5F = (1.06 ~ 0.04)? = (1.02)? = 1.0404 66 (a). Let the total score be x runs, euch that x(- or, 3rd = 8 or, x = 162 67 (a). (4) + (68)? 4-32)" 1+ ("2 + 1x 32) 1+ 81 + (CI « G2)" 1481+ (CPPS & 2") + tag x ig eit Leg tU x 16) 7d 4 2 -9x 8x1 68 (by, x = (OE=9e121, _ Bet 12164 (8)? 43x11 (64) = 3x91, 8x11 Tixdetxk “64033 on 70 (a). LCM. of 7, 8, 16 and 35 = 560 5x70 _ 350 8x70 ~ 360 21x16 _ 336 35x16 560 9x35 _ 315 16x35 ~ 560 6x80 _ 480 7x80 560 2.2 xox =8 99 ‘Simplication 8 4.21 Difference between the largestand the smallest fractions nS Fg M00 AIS 16S = 38 7 16 560 560 ~ 560 112 74 (b). Let the money with the man at first be Re | 5 =RG 5 Money spent = 2 of 1 5 Remaining money = 1 = 3 = Ref 7 dat ofesterst and money eamed = + of Rs = R65 Total money with him now aia 3 1 6a eg ‘La part of his money is with nim now 72 (d). Let the monthly income of Manmohan be Re 1 4 Saving = = 3 = Monthly income 73{c). Let the number be 1 4 Zofl 4 Difference = = ~ 1 Number = 4 + 55 rate, 2 3 na 3 33 Part = 96 « 5x5 = 108 274 75 (b). Joumey completed by aeroplane and trein . 246 8 ~ is ~ 15 Remaining joumey ecole 1 pein ys stra) ( aa You have either reached a page that is unavailable for viewing or reached your viewing limit for this book. aa You have either reached a page that is unavailable for viewing or reached your viewing limit for this book. aa You have either reached a page that is unavailable for viewing or reached your viewing limit for this book. aa You have either reached a page that is unavailable for viewing or reached your viewing limit for this book. 426% Quantiaive Aptitude 75 (b). 76 (c). 77 (e). x 2h - 2 = y= 48 = ky = +2)? 78 (c). Suppose there were x packages in the van before delivery, After Ist delivery, the number of packages 3 in the van = x s*"s After 2nd dolivery, the number of packages in Br-15 79 (a). (6% 5° = 216 = 5 = 1080 (8x2) 12x 2= 1024 83(c). Let, 84 (d). 85 (b). (2130)? = 8 « 130= 1040 ¥900)° = 900 80(b). x= 2+ = = (x2 = +2" 44243 «229219 [298 4219) = 643% 2(¢-2) => (e- 2) =6 + Gr - 12 = 6r~6 > 8 bxle2)~ 6x6 = xo -6r+ br =2e4 (a? + ede a +h) This becomes a quadratic equation in x and. when discriminant, D 20, we get the result a) 1003 AOOK) 1083 (999 }” 1001 EXERCISES FOR SELF-PRACTICE Exercise 4.3 0 @ 0 1 107 (©) 100 () 505 The value of 1 + 2 he value of + 75 TTTy 2 places of decimal is (164 () 165 © 19 @ 18 detort 3°3"3_, 1a al 35's Level I @ 5 1 2 Og (@ 28 L Ina certain office, ~ rd of the workers are women, 3 1 1 Grd of the women are married and = rd of the married and 2rd ofthe maried men have children, what part of the workers are without children? 5 4 rs Os u 7 ie © 5 @ © aa You have either reached a page that is unavailable for viewing or reached your viewing limit for this book. aa You have either reached a page that is unavailable for viewing or reached your viewing limit for this book. aa You have either reached a page that is unavailable for viewing or reached your viewing limit for this book. 52 1 Quantitative Aptinde ane eeacn Chena en ntae i The following table gives percentage values of some frequently used fractions Table S.1 NUMERATORS oan oa Git 7 8 9 if eos} aa fg, 1 }100 | 207 1300 | 00 | so | oo m0 90 | 900 | 1000 | 1100 | 1200 2}s0 }100 |1s0 | 00 || 250 | 300 mo 40 jo | 7 | on 3 (3333 | 6666 | 100 | 13833 | 16666 | 207 23833 | 24666 | 30 -8BS | 36656 | 400 gla ja 9 | | | as in 1s aw 0 | 78300 E sm» o )o wm | wo m2 M8 Z| 6 | 16.66 50 66.66 | 83.33 100 116.66 133.33 . 150 166.66 | 183.33 200 S\7 | 1428 42.85 maz | 8571 | 100 414.28 | 128.56 42.85 157.13 142, Bsns ws | 50 [os [1 | es 0 125. ms 372s | 150 Blo jun Mat | 5855 | 66067177) BRR Ma? IB wo 20 o |0 |@ |» » 9% | 10 | 0 120 ur [900 | isis | 2727 | 3636 | 454s | 5454 636 7272 | ssl 9 | 100 10909 12 {ax | 1666 | 25 anos | 305833 833 91.66 100 is [66s | 1333 [20 |} 2566 | 3333 40 | as00. 15333 or | 6566 | 7333 80 Learn and practice all the values given above SOME USEFUL SHORT-CUT METHODS SC L. (a) If A is.x% more than that of B, then B is less than that of 4 by [wads 100} (©) If 4 is.x% less than that of B, then B is more then that of 4 by (aczxim9] in Explanation Given 4 100+ ven ae Anb o-8 12s) )g 100 10? Tore, ~ Toor 700 = A- —<_ x 100] 9% of Therefore, B is less than that of A by (sahzx100) Twos Similarly, (b) can be proved. Iilustration 6 If Mohan's salary is 10% more than that, ‘of Schan, then how much per cent is Sohan’s selary less, than that of Moban? Solution Here, x = 10 Rosia anor = (caso) a 1004. = (lt prio) bsg 3 Mustration 7 If.4’sincome is 40% less than B's income, then how much per cent is B's income more than 4’s Solution tere, x = 40 Required answer = (—*— 100} ‘ (woz) = (a niem)pe 100-40 662% 3 SC 2. IFA is x% of C and B is y% of C. then t= ~ + 100% of B aa You have either reached a page that is unavailable for viewing or reached your viewing limit for this book. aa You have either reached a page that is unavailable for viewing or reached your viewing limit for this book. aa You have either reached a page that is unavailable for viewing or reached your viewing limit for this book. 56 ® Quanutaive Apuude SC 9. In en examination, the minimum pass percentage is x%, If a student secures y marks and fails by ‘2 marks, then the maximum macks inthe examination SC 10, In an examination x% and y% students respectively fail in two different subjects while 2o students fail in both the subjects, then the 100(»-+2) percentage of students who pass in both the ee subjects will be (100 ~ (x + y — z))%. Explanation Explanation Let the maximum marks bem. Given: x% of m=y+z x 1o0(y +2) 100 x Mlustration 21 In an examination, a student must get (60% marks to pass. If a student who gets 120 marks, fails by 60 marks, find the maximum marks. Solution Here, x= 60, y = 120 and z = 60. ‘age of students who failed in one subject only (x - 2)% aye of students who failed in other subject only 2% ‘Sage of students who failed in both the subjects = 29% % age of students who passed in both the subjects, = [100 = [(x = 2) + = 2) + 21% = (100 ~ (x+y — 2% Mlustration 22 In an exsmination, 42% students failed in Mathematics and 52% failed in Science. If 17% failed in both the subjects, find the percentage of these who passed in both the subjects Solution Here, x = 42, y = 52 and 2 = 17 Percentage of students passing both the subjects. = (100 ~ (@ +y—2))% = (100 ~ (42 + 52 17% or 23% MULTIPLE-CHOICE QUESTIONS axinam mss = 1000) , OKI20+) 100% 180 _ 60 Exercise 5.1 1 1. What percentage is equivalent to 55 (a) 525% (b) 425% (©) 625% (@) None of these 20-008 = 6.9 3% 1, 1, © 4% @) 5% © % (@) None of these 1 og oF (@ None of these 20 4. What fraction is 0.6%? os oe 500 300 os (@ None of these 5. 0.025 in terms of rate per cent is (a) 3.5% (b) 2.5% (©) 15% (@) None of these 6. What per cent of 12 is 84? (a) 800% (b) 600% (©) 700% (@) None of these 2. Expres 7 a perce 1 win w 75 © 9 (©) None of these 8 Express 83% as a fraction. 1 1 Ord O % ne 18 © (@) None of these aa You have either reached a page that is unavailable for viewing or reached your viewing limit for this book. aa You have either reached a page that is unavailable for viewing or reached your viewing limit for this book. aa You have either reached a page that is unavailable for viewing or reached your viewing limit for this book. ive Aptitude ) (a) 40% 70. Ina fraction, numerator is increased by 25% and the denominator is diminished by 10%. The wie 5 traction obtained is >. The original fraction was 5 a (a) 3 OF Exercise 5.2 A person has some amount with him. 25% of tisstolen ina bus, 10% islost through a hole in the pocket, 50% of remainder is spent on food. He then purchases a ‘book worth Rs 26 from the remainder. He walks back home because all his money is over. What was the initial amount? (a) Rs 160. (©) Rs90 (by Rs1.230 (@) Rss Three papers where set in an examination and the maximum marks per paper were in the ratio of 1 :2 2 respectively. If a student obtained 30% in the fist psper, 60% in the second, and 65% in the third, what per cent did he obtain overall? (a) 58.3% (b) 66.66% (©) 33.33% (a) 60% OF the total number of candidates 40% fal in frst subject, ofthe est 10% fil in the second, and of those that pass in these two subjects, only 75% aan examination pass in the remaining subject. The number of unsuccessful candidates exceeds that of the suecessfal cones by 570. What is the total number of candidates? (a) 2800 () 8400 (3.000 () 120 Ram’sweight is 140% of Manu’s weight. Tanu’s weight is 90% of Mahesh’s weight. Mahesh weighs tw much as Manu, What percentage of As is Tanu’s weight? (a) 64.3% (by 77. fe) ry (a) 1286% A businessman's eaming increases by 25 but decreases by 4% in the next, After S years his total earnings would be Rs 72,000, What is his present (a) Rs10,000 (b) Rs 80,000 (©) Rs40,000 (a) Rs54.000 Arman invests Rs 1.200 at 10% pa, At the end of the year he withdraws 30% of total amount and pays Rs 24 as transaction fee, At the end of 2nd year he n. 10. @2 5 (One side of a square is increased by 30%. To maintain the same area the other side will have to be decreased by (d) None of these abe 12, @ 23% 0) 165% © 3% (d) 19% Level II withdraws 30% of the amount and pays Rs 93 as transaction fee. What is the balance at the end of the third year? (a) RS660 (b) Rs $25 (©) R500 (a) Rs770 A family’sratioof savings to expenditure for last month was 2: 13. This month, duc to unforeseen expenditure, savings fell 10 50% of the amount saved last month. Salary, fast month was Rs 10,000/-, This month there was an increase of 15% in the salary. How rmuch did the family spend this month? (@) Rs667 (b) RSi1,167 (©) Rst0833 (a) Rs 9,833 500 kg ofore contained a certain amount of iron. After the first blast furnace process, 200kg of slag containing 12.5% of iron was removed. The percentage of iron in the remaining ore was found to be 20°% more than the percentage in the original ore. How many kg of inon were there in the original 500 kg ore? @ 224 () 83 7 (by 892 (@) 145 6 Ua > b, then by what percentage is less then its a reciprocal? bot w wfS-t) rol wmf-8) ms A trader sells soaps in economy packs of four soaps per pack, each pack being charged at the listed price of three soaps. For every set of five such packs bought by a customer, the trader gives him one extra soap as afree gift Ifa customer buys 12 economy packs, what is the effective percentage of discount that he gets? aa You have either reached a page that is unavailable for viewing or reached your viewing limit for this book. aa You have either reached a page that is unavailable for viewing or reached your viewing limit for this book. aa You have either reached a page that is unavailable for viewing or reached your viewing limit for this book. 5148 Quantiative Apsitude 50% of Vikas's weight. Vikas weighs 190% of Mayur’s weight. Which of these people weighs the least? a) Mayur (b) Deepika () Shweta (d)_ Rakesh from I to 70 have 40. What percentage of number squares that end in the digit 1? (a) 20 (by) 1 2 41, jay and Vikas are sharing a flat in Delhi, with an arrangement of equally dividing the household expenses. Ajay went to Pune, wherea sale was going on and bought batteries for the house, worth Rs 150, on 20% discount. But he lost them on his way back and had to buy new ones, after he reached: Delhi How much did he end up spending on the batteries? (a) Rs 280 (b) Rs 195 (©) Rs 270 (a) Rs 75 Directions (Questions 42 to 45): 1t is given that 5% increase in V always means 3% increase in Y and increase in Y always implies 2.5% increase in Z, 42. Relationship between X and Z could be (a) 1.05¥ = 1.025% (b) OSX = 0.257 (c) 2S¥= 752 (a) 250X = 2132 43. If Vis increased by 30%, then 2? should increase by (a) 32.25% (0) @% (e) 69% (@) 0% 44. If X is increased to 23 from 20, then ¥ should increase from 100 to (a) 103 (b) 106 fe) 109, @ U2 45. If X'ig increased from 10 to 13, then Z must increase from 30 to (a) 35 (b) 45 ©) 00 @ 5 46. Salary of an officer increases every year by 20%. His salary in the year 2001 was Rs 26,649, What was his salary in 1999? fa) Re 20,000, () Rs 19,028 (©) Rs 18,500 (a) Rs 18840 47. The owner of a restaurant decides to calculate the percentage of vegetarian Jain dishes sold. If 70 per cent of the dishes sold are vegetarian and of this 20 per cent are Jain dishes, then what percentage of the toial dishes sold are vegetarian Jain dishes? (a) 50% (b) 18% (©) 20% (@) 14% 48." In a certain shop, which stocks four types of caps, there are 1/3 as many red caps as blue and 1/2 a8 many green caps as red caps. There are equal numbers of green caps and yellow caps. If there 9. sh. 3. 54. 37. are 42 blue caps, then what per cent of the total caps in the shop are blue? (a) 70% () 28% {o) 60% (a). 14% ‘A bag contains 600 pens of brand 4 and 1,200 pens of brand B. If 12% of brand A pens and 25% of brand B pens are removed, thon what is the approximate percentage of total pens removed from the bag? (a) 37% ©) 36% (o) 22% @ 18% A invested 125% as much money as B. C invested 80% as much money as B. The total of all the three is 61,000. How much did C invest? (a) 25.000 (6) 16,000 (©) 2000 (@) 45,000 A retailer buys a stereo for Rs 225. His overhead expenses are Rs 15, He sells the stereo for Rs 300. ‘What percentage return on this cost did he get on this transaction? (a) 10 (b) 25 (©) 33 (a) 50 AA shopkeeper offers to sell 334% more quantity of a product for a price that is 20% higher. What is the effective discount that he is offering? (a) 10.8% (b) 16.67% (c) 13% (@) 10% The price of cooking oil has inereased by 25%. ‘The percentage of reduction that a femily should effect in the use of cooking oil so as not to increase the expenditure on this account is (a) 25% (0) 30% (©) 20% (@) 15% A flat costs Rs 10 Jakh, Incidental expenses. and taxes amount to Rs 36,000 a year. What rent per month must the owner receive to carn at Ieast 6 per cent on his investment? (a) Rs-6,000 (b) Rs 6500 (©) Rs 8,000 (@) Rs8500 A businessman charges 20% over cost price. He allows his customers 10% off on their. bills for cash payment, His net gain per cent is @ 10 ) & (©) 12 (@ 1s X gets 25% more than ¥ and ¥ gets 20% more than Z. The share of Z out of a sum of Rs 740 is (a) Rs 200 (b) Rs 300 (©) Rs 250 (d) Rs 350 ‘A man's wage was reduced by 50%. The reduced wage was then increased by 50%, His loss in percentage term is aa You have either reached a page that is unavailable for viewing or reached your viewing limit for this book. aa You have either reached a page that is unavailable for viewing or reached your viewing limit for this book. aa You have either reached a page that is unavailable for viewing or reached your viewing limit for this book. 5.18 © Quantitaive Apuinude and 20% of unskilled workers are permanent. If the number of temporary workers is 126, then what is the number of total workers? bearings were defective. If the company received a total of 100 defective ball bearings, how many ball bearings were in the first shipment’ @ 480 ‘e510 @ 2,000 () 1,000 (c) 360 (d) 377 (©) 990 (d) 3,000 102. Ontime % 104, A clothing suplier stores 800 coats in a warehouse, Upto 15 minutes delayed 43% Of which 15 per cent are full-length costs, If SU0 15.30 minutes delayed 17% of the shorter lengih coats are removed from the 30-60 minutes delayed 12% warehouse. what per cent of the remaining coats is ‘More than 60 minutes delayed % full-length? The chat above describes om aceruinarponton 3% yaa a certain day. If 1,200 flights were delayed, how (©). 5.62%, (ay 40% many fights deparied on time? 105, In a ity, 35% of the population is composed of @ 0 om migrants, 20% of whom are from rural areas. OF the (c) 250 (d) 400, Jocal population, 48% is female while this figure for is 30% and 4% 193. A company received two shipments of ball bearings ealand wn migrot MPC unl 40% repel) Ifthe total population of the city is 7.28400, what In the first shipment, | per cent of the ball bearings is its female population? were defective, n the second shipment, which was aon een twice as large as the first, 45 per cent of the ball oo Oe dew ANSWERS Exercise 5.1 Level I L@ 2% 23 £0 5M BO 7M 8 2 ©) mo moO mo Oo hem oO mo no ko Bm Mo Ww BO Bo MO B&H % WH MO Mo Mm RO Bo) OM RO) OH OF HH) 1) 2 © RO Hh Oe O&O we © me @ © oOo 2 © % oe & w & O 2 & ) BW & OH HH & © Bm & oO 6% 6%) © & & O&M Mw Th @ Exercise 5.2 Level II L@ 2% © © £0 56 &£@ 270 & Oo 26 © o hwo RO BO HO Bw Hw OH n@ & @ mM & Exercise 5.3 Questions from Competitive Examinations Lo 2m 2m &£H@ FO FHM 2H RO 2% 8H © O Rw MO HO K @ 6 w aa You have either reached a page that is unavailable for viewing or reached your viewing limit for this book. aa You have either reached a page that is unavailable for viewing or reached your viewing limit for this book. aa You have either reached a page that is unavailable for viewing or reached your viewing limit for this book. 522 0 Quintiaive Aptinude lox 10 = (-tor10- [Here x = -10 and y= 10] =-1% The revenue decreases by 1% 44 (a). Since 4p radius * radius = surface area Net change inarea { cys 1% 45 (b). We have, receipts = price * sales Net % change in receipts = 42% 4 46 (b). Since side * side = area Net % change in arca = (204206 ae 100 = 6% The area is increased by 69% 47 (a). ince side, * side, = area Net % change in area 100 ~ (204204 se 100 [Here, « = 30 and y= 20] = 56% ‘The area of the rectangle so formed exceeds the area of the square by 56% 48 (a). Since side, * side, = area wo) nor 9 in aea=(s-+y+2% Jos 2 ( ”Ta5) -(10-20 120), ii [Here,x~ 10 and y 49 (c). Since side, * side, Net % change in area ‘The area of the rectangle increases by 32! 50 (a). Since tax consumption = expenditure Net % change in expenditure ( wy), « [n-an-222), = Expenditure decreases by 4% 51 (a), Net % change in revenue { 0 og = [ery +2 Jos, [+»*i00) 30+ 20: 20220), 100 [Here, x= 30 and y 16% 52 (b). We have, P= 90.51, r = 10 and» The population 3 years ago P 90-51 (es) (v2) U7 00, 100 901,100, 100, 100 100 “110 "110" 110 53 (c). Here, P= 8748, r= -10 and n= 3 Purchase price of the machine 8748 bee) fr 100 8748 1005100 100 90%90%90) 54 (c). Here, P = 2664000, r= 20 and n= 2 ‘Company's income in the year 1997 P____ 2664000 = Rs 12,000 aa You have either reached a page that is unavailable for viewing or reached your viewing limit for this book. aa You have either reached a page that is unavailable for viewing or reached your viewing limit for this book. aa You have either reached a page that is unavailable for viewing or reached your viewing limit for this book. 526 Quantitative Aptitude 1(b). 2 (p). 3 (a). 4(d). 7 (b). 8. (c). 9 (b). 11 (c). Exercise 5.3 Questions from’ Competitive Examinations 14 out of 200 machine parts were defective, Le. 200 _ 1605 1% 2 100» 105 110.25. = Mes sx-16 Let x% of - (220) xi-15) 412 (b). (20) + (15) + POEMS) _ gy Required ratio (b). (-20) + 15) a 13 (c).. 15% = 60000. 75% ~ 300000 where d is density of the substance =Total amount spent on machinery and Given 117d, = I5ldy raw material dy _ 151 tae. 5 ar @). 3 .{ 25.1600) - rs 20 d, 17 ( v09 {iso ) Now with 7¥of substance 8, AY, of subsane® 45) 94 of y 4 is used in place of 4¥', 134 = ny = terror = 4M 190 w 22% U7 15st 16 (a). 100 ~ 20% = 80 . = 1S alos GE: 17 (b). Let Mr X’s salary = Rs 100 Suppose the number of bushes in the experim- Salary after inerease = Rs 120, is.. Rs 20%8 ental field = x the increase in salary on Rs 100 Nomber of Uushes’afer one year: Let the tax on the original salary was 3 te Hence the tax on the increased salary (Rs 20) = + 10% ots = TE will be 33%, i.e. Rs 6.60 Number of bushes after 2nd year “2. Imerease in tx tabitty - = «190 = 22% iy Mx | 297 ° = DE goof UE “io 89 To 350 18 (c). Let reduced price of the radio be Rs x Number of bushes alter 3rd year e+ 7% of e = 2568 => 107% = 256800 BIE _ 9x5 of 227 = r= 2400 2A Reduction needed inthe price of radio = Rs 168 2500" 19(c). 600 — 2673. = 26730 = x = 25,000 sen 2500 20 (a). mth member contributed Rs (10 * 2" ~ 5) A=B+25% of B => 1st member contributed Rs 15 gene! 2nd member contributed Rs 35 4 3rd member contributed Rs 75 3 40% ota 44th_ member contributed Rs 155 Ard so on Let the original price be Rs © per kz Since 4th member gets Rs 62 as his share in the 4 profit, therefore we conleude that 40% profit is Reduced price ~ Rs 4s per ke ceamed by each member. 4 . Total profit earned => fx 10 Wot = 40% of [15 +35 +75 +... + upto 100 terms] [3+ 7+ 19+31 + .. + upto 100 terms} aa You have either reached a page that is unavailable for viewing or reached your viewing limit for this book. aa You have either reached a page that is unavailable for viewing or reached your viewing limit for this book. aa You have either reached a page that is unavailable for viewing or reached your viewing limit for this book. 530 © Quantiative Aputude 69 (c). Let the original safe price by 10 units each ‘Then, original revenue collection = 10 * 10 = 100. New price= 10 13 = 13. New safe = 10%-8=8 New revenue collection ~ 104. Hence, revenue is increased by 4% 70 (d). Let number of men and women be 40 and 60 respectively. «= Number of men eaming more than Rs 25,000 =30 ‘Total number of employees earning more then Rs 25,000 45 = Number of women eaming more than Rs 25,000 = (45 ~ 30) = Rs 15 Now fraction of the women earning Rs 25,000 or less _-15_ 45 3 0 60 71 (@).. Total number of characters =20* 55 65 = 71500 Number of pages required, if the report is retyped 71300 exo” 170 Hence, 16 pages are required. Hence % reduction 20-16 7 720% 72 (c). Increase in price of sugar = (8 +2) = 10% Price of sugar on January 1. 1996 20 1 * LL = Rs 24.20 73 (c). Let x voters voted against the party in the Assembly Poll. Then votes in favour = ( ~ x) (For calculation simplicity suppose 2,60,000 = ») Majority of votes by which party won in Previous poll =@-H-3=6-29 Now, votes polled against the party in general election = 1.25 x and votes polled in favour of the party (r= 1.25%) Majarity of votes by which party lost is general election = (1.25 x) 125) =(2.5x-y) It is given thet, (25x — y) = 2 ~ 2x) = 25x ~ y= 29 Ay 3 2,60,000 oF 65x=3px= = = 120.000 Votes polled by the voters for the party in Assembly polls = (260,000 ~ 1,20,000) = 1,40,000 74 (a). Let x be the number of people who were asked for donation People already solicited = 0.6x Remaining people = 0.4x Amount collected from the people solicited 75 (a). 76 (c). = 6000 « 6.0 = 360% which is 75% of the amount to be collected. Remaining amount 24 Average donation from remaining people 120% ~ ax 08 Let the number of questions in 4, B and € be @, 6 and e respectively. We have a +h +c = 100. Total marks would be a + 2b + 3¢ Given 6 = 23, total marks from section B = 46 Different possible values for ¢ are 1.2, 3, Corresponding values for aare 76, 75, 74, (since total questions are 100) When ¢ = 1,a= 76, 6 = 23 Toval marks from groups 4, 8, Care 76, 46, 3 respectively sie oa Percentage marks from 4= oy Bs 60% 125 When ¢ = 15, b= 2 p 3s ercentage marks from A= see = Bow 127 Forall other values, when c inereases, a decreases and contribution of marks from 4 keeps decreasing There is only 1 possible value for questions feom group C Marks from group C = 24 Since B contributes ot least 20% and A contributes at least 60% to the total contribution from C is maximum of the 20%. Total marks 2 120, Total questions are 100. So a+ = 72 Minimum number of questions in A = 72, and in B= 12 Bal seace!%%, cout © 6 aw) (at20. bution oF +39 4 sizso 20 256m 0 sin ww 60" 20 $1478 130 con. > 20% | os eis? ut 6 > 20 Hence, only 2 possible values of b exist aa You have either reached a page that is unavailable for viewing or reached your viewing limit for this book. aa You have either reached a page that is unavailable for viewing or reached your viewing limit for this book. aa You have either reached a page that is unavailable for viewing or reached your viewing limit for this book. 534 Ex © Quanutave Apamde cercise 5.5 If company *P" were to produce 400 cakes more per day than the present quantity, then, assuming that the percentage of cakes ofthe different varieties produced by company ‘P" is the same, what will be the approximate percentage of the total number of cakes ‘of (B) variety produced by all the companies per day, to the total number of cakes-produced by all the ‘companies per day’? (a) 31% tb) 25% (e) 29% (d) 33% Parmeet gets 112 and fails his test by 28 marks. Ifthe ‘minimum percentage required to pass the testis 40%, ‘what are the maximum marks in the test? a) 320 (b) 350 (e) 3 (@) 360 The weight of 4is 4Skg. 4 is 60% of Band ihe weight of the sum of the weights of 4 and 8. ight of Bis increased by 334% , what will be the percentage increase in the total weight of A, Band C's the weights of 4 and C increase soas 10 ‘maintain the proportion they had with & earlier?” l a) 5% 162! @5 (b) 165 1 fe) 334% id) 25% © 35 (ad % 4. Ramesh hadsome oranges Outof that, 4% were thrown, away. 80% of the remaining oranges were soll and he is now teft with 96 oranges, The initial number of ‘oranges with him was (a) 450 (b) 500 (e) 350 (a) 500 5. Inastockpile of products of three machines, 40% 30% ‘and 30% were manufactured on machine I, mackine It and machine III respectively, 3% of the products of ‘machine I are defective, 1% of products of machine Il ANSWERS Exercise 5.4 1b) 2 ©) 3 6) 4 @) i) mo Exercise 5.5 o 2 (b) 20 40 9% (a) 1, by EZ Level TI are defective while 95% of the products of machine It are non-defective, What is the percentage of defects in the stockpile? (a) 3 (b) 9% (o) 15% (d) 30% Ifthe side of a smaller square is 20% less than that of bigger one. then what percentage more is the area of er square as compared to smaller one? (a) 36 tb) 65% (©) 56% (a) 50% By how mach percentage the consumption of sugar be decreased so that 22% imerease in the price will be nullified? (a) 18% () 16.67% Directions (Questions 8 to 10): Kead the following information carefully and answer the questions that follow The values of a, bande ate 20, 1S and 20, Ifais decreased by 2.5%, bis decreased by 8 10, 13, “sand ¢ is decreased by 2% then: (b) 2% (a) 2.7% (b) 2.7%. (a) 7.2% Ifthe values of a, b and ¢ are changed to 200, 150, and 400 respectively instead of the ones given above, (ax) then after the respective decrements as mentionedabove will be ( 75 (b) 71.63 (©) 70.52 (@ 70 Leyel I © 6) 7.) & om @) a @ 15. (a) Level I @ & © 7 @ & aa You have either reached a page that is unavailable for viewing or reached your viewing limit for this book. aa You have either reached a page that is unavailable for viewing or reached your viewing limit for this book. aa You have either reached a page that is unavailable for viewing or reached your viewing limit for this book. 64 ¥ Quuntstiye Apuinde Mustration 17 Find the average of 7 consecutive numbers 3, 4, 5, 6, 7. 8, 9 Solution The required average = middle number = 6 Mlustration 18 Find the average of consecutive odd numbers 21, 23, 25, 27, 29, 31, 33, 34 Solution The required average = average of middle two numbers = average of 27 and 29 Mlustration 19 Find the average of first 31 consecutive even num Solution ‘The required average ~ (1 N=31+1=32 Mlustration 20 Find the average of first 50 consecutive odd numbers, Solution The required averag Mustration 21. Find the average of squares of first 19 consecutive even numbers. 50 2+ YQn4h 3 D4 DE Solution ‘The required average 3 2x20x39 1560 _ 559 3 Mlustration 22 Find the average of squares of consecutive even numbers from 1 t0 25 Solution The required averaze (enor?) _ 25402: ~ 3 6x27 _ 702 33 = 234 Illustration 23 Find the average of squares of consecutive odd numbers from 1 to 31 Solution The required average nat2) _ 31XG1 3 31x33 +2) ML Mlustration 24 If the average of 6 consecutive numbers is 48, what is the difference between the smallest and the largest number? Solution ‘The required difference 1 = I= 6-1) = 10 SC7. Geometric Mean or Geometric Average Geometric meen OF 2,..%3, sna 4, iS denoted by GM. = Yr eax IMPORTANT Geomewie mean is useful in caleulating averages of ratios such as average population growth rate, average percentage increase and so on. Wlustration 25 The production of a company for th sive years has increased by 10%, 20% and 4 ely. What is the average annual increase of produ tion’? Solution Geometric mean of x, yand == (ry * 2)! Average increase = (10 * 20 « 40)'% = 20% Mustration 26 The population of a city in wo successive years increases at the rates of 16% and 4° respectively. Find the average inerease of two years Solution In case of population inci mean is required, Geometric mean of | (a6 « 4)" e. the geometric fa and ie. 8 SC & Harmonic Mean or Harmonie Average Harmonic means of x), 3, -, tis denoted by HM. = —— IMPORTANT Harmonie mean is useful for finding out average speed of a vehicle, average production per day and so on, Mustration 27 A man runs I km at 15 km per hour and another | km he walks at S km per hour. Find his average speed for the whole distance in covering 2 km. Solution When the distance is constant and the speed varies, harmonic mean is used. Harmonic mean of x and, Average speed for the whole distance 15x 1545 = 7.5 km/h SC9, Ifa certain distance is covered ata speed of x kmvh and the same distance is covered at a speed of y kmh, the average speed during the entre journey is aa You have either reached a page that is unavailable for viewing or reached your viewing limit for this book. aa You have either reached a page that is unavailable for viewing or reached your viewing limit for this book. aa You have either reached a page that is unavailable for viewing or reached your viewing limit for this book. 6a 38. 36. 7 39, a. 42, a. Quantiative Aptitude by 10, the expenses of the mess were increased by Rs 40 per month while the average expenditure per head diminished by Rs 2. Find the original monthly expenses. (@) Rs 390 (b) Rs 410 (©) Rs 360 (d) Rs 480 Of the three numbers the first is twice the second and the second is thrice the third. If the average of the three numbers is 10, the numbers are (a) 18, 3,9 (b) 9. 3.18, © 3,9, 18 (d) 18, 9,3 The average weight of 36 students is 50 kg. It was found later that the figure of 37 kg was misread as 73 kg. What is the correct average? (a) 49 kg (b) 51 kg. (©) 50.5 kg. (a) None of these ‘The average eaming of a mechanic for the first four days of a week is Rs 18 and for the last four days is Rs 22. If he earns Rs 20 on the fourth day, his average earning for the whole week is (@) Rs 18.95 (b) Rs 16 (©) Rs 20 (@) Rs 25.71 The average of marks obtained by 120 candidates ‘was 35. If the average of marks of passed candidates, was 39 and that of failed candidates was 15, the number of candidates who passed the examination is (a) 100 (b) 110 (©) 120 (@) 150 In a class, there are 20 boys whose average age is ‘decreased by 2 months, when one doy aged 18 years, in replaced by a new boy. The age of the new boy is (a) 14 years 8 months. (b) 15 years (©) 16 years 4 months (d) 17 years 10 monttis ‘The average temperature from Monday to Thursday is 48° and from Tuesday to Friday is 52°. If the temperature on Monday is 42°, what was it on Friday? (a) 52° {b) 55° © 58° (a) SIP A man spends on an average Rs 269.47 for the first 7 months and Rs 281.08 for the next $ months. Find his monthly salary if he saves Rs 308.46 during the whole year. (a) Re 400 (b) Rs 500 (©) Rs 300 (a) Rs 600 The average of two numbers is 62. If2 is added to the smaller number, he ratio between the numbers becomes 1:2. The smaller number is (@) 60 (b) 30 {c) 84 {d) 40 In a coconut grove, (x + 2) tres yield 60 muts per year, x trees yield 120 nuts per year and (x ~ 2) 44. 45. 46. 47. 48, 49. 51. trees yield 180 nuts per year. If the average yield per yeor per tree be 100, find @4 (b) 2 8 (a) 6 Average temperature of first 4 days of « week is 38.6°C and that of the last 4 days is 40.3°C. If the average temperature of the week be 39.1°C, the temperature on 4th day is (a) 367°C (b) 38.6C (©) 398°C (a) a1.C The average daily wages of 4, B and C is Rs 120. If Beams Rs 40 more than C per day and 4 earns double of what C earns per day, the wage of 4 per day is (a) Rs 80 (©) Rs 160 (b) Rs 120 (2) Rs 100 With an average speed of 40 knvh a train reaches its destination on time. If it goes with an average speed of 35 kmh, it is late by 15 minutes. The total journey is (a) 30 km (b) 40 km (©) 70 km (@) 80 km In a competitive examination, the average marks oblained were 45. It was later discovered that there ‘was some error in computerization and the marks of 90 candidates had to be changed from 80 to 50 and theaverage came down to.40 marks. The total number of candidates who appeared in examination is (a) 520 (b) 550 (©) 540 (@) 535 Visitors to a show were charged Rs 15.00 each on the first day, Rs 7.50 on the second, Rs 2.50 on the third day and total attendance on the three days were in the ratio 2: $: 13, respectively. Find the average charge per person for the whole show. (a) Rs7 (b) Rs 5 (©) Rs9 (@) Rs 11 The mean daily profit made by a shopkeeper in a month of 30 days was Rs 350. If the mean profit for the first fifteen days was Rs 275, then the mean profit for the last fifteen days would be {a) Rs 200 (b) Rs 275 (©) Rs 350 (@) Rs 425 A man whose bowling average is 12.4 tkes 5 wickets for 26 runs and thereby decreases his average by 0.4. The number of wickets, taken by him, before this last match is (a) 85 (b) 78 (©) 72 (d) 64 Out of three numbers, the first is twice the second and is half of the third, If the average of the three numbers is 56, the three numbers in order are aa You have either reached a page that is unavailable for viewing or reached your viewing limit for this book. aa You have either reached a page that is unavailable for viewing or reached your viewing limit for this book. aa You have either reached a page that is unavailable for viewing or reached your viewing limit for this book. 6.12 © Quantitative Aptitude 17. 18, 19, 20, 21 22, 23, 24, (a) 24 years (©) 28 years (b} 27 years (@) 30 years The average of thtee numbers is 135. The largest number is 180 and the difference of the other numbers is 25. The smallest number is, (a 130 (b) 125 (© 120 (¢) 100 The ratio of the arithmetic mean of two numbers to ‘one of the numbers is 3 : 5. What is the ratio of the smaller number to the larger one? @ 1:5 (b) ts4 1:3 @) 1:2 A person travels from X to ¥ at a speed of, 40 kmph and returns by increasing his speed by 50%. What is his average speed for both the trips? (a) 36 kmph (b) 45 kmph (0 48 kmph (4) 50 kmph ‘A painter is paid x rupees for painting every 10 metre: of a wall and y rupees for painting every extra metre, During one week, be painted 10 metres ‘on Monday, 13 mettes on Tuesday, 12 metres on Wednesday, 11 metres on Thursday and 12 metres on Friday. What is his average daily earning in rupees for the five day week? (a) x > (8/5)y (0) (Se + 9y/3 (©) lox + (8/5)y (4) Sx + The average of S consecutive numbers is n. If the next two numbers are also included, the average will (@) Increase by 1 (b) Remain the same (©) Increase by 14 (a) Increase by 2 For ten hours, a train travels ata constant speed of 20 miles per hour and during the next 15 hours, it travels 240 miles, What is the average speed of the train for the whole journey? (@) 176 milesshour (6) 20.8 milesshour (6) 130 miteshour—(€) 176 miles/hour Average of ten positive numbers is T. If each number increases by 10%, then (2) Remains unchanged (b) Is increased by 10% (©) May decrease (@) May either inerease or decrease On an 800 miles trip, car W travelled half the distance a 80 miles per hour and the other half at 100 miles, per hour. What was the average speed of the car? (@) 18.00 (b) 180.00 (©) 99.00 (a) 885 28, 26. 22. 28, 29, 30, 31. 2. Having scored 98 rans in the 19th innings, a cricketer increases his average score by 4. What will be his average score after the 19th innings? (a) 8 (b) 26 ©) 44 (¢) 22 ‘Angad was conducting an experiment in which the average of 11 observations came 10 be 90, while the average of first five observations was 87, end that of the last five was 84, What was the measure of the 6th observation? (a) 165 (b) 150 (©) 145 (@ U5 A student who scored 30% marks in the first paper of Physics out of 180 marks, has to get an overall score of at least 50% in two papers, the second paper carrying 150 marks. What percentage of marks should he score in the second paper to get the overall average score? (a) 80% (&) 76% (©) 74% (d) 70% Tnan examination, apuril’s average marks were 63 per paper. If he had obtained 20 more marks for his, Geography paper and 2 more marks for his History paper, his average per paper would have been 65, How rmany papers were there in the examination? (a) 8 () 9 (©) 10 1 During a season, a batsman played 20 innings. In the first four innings, he scored an average of 20.8 runs; in the next six, he scored an average of 33 his; in the next seven innings, he scored an averege of 17.5 nuns, and in the last thres, he scored nothing, ‘What was his average per innings for the season’ (@) 20.19 213 © 39 (@ 23.95 Three math classes: X, ¥ and Z, take an algebra test. The average score of class X is 83 ‘The average score of class Y is 76 The average score of class Z is 85. What is the average score of classes X, ¥, Z? @ a5 (b) 80.5 ©) 3 (6) In three numbers, the first is twice the second and thrice the third, Ifthe average of these three numbers is 44, then the first number is fa) 12 (b) 24 fe) 36 (a) 44 ‘The batting average of 40 innings of a cricket player is 50 runs. His highest score exceeds his lowest score by 172 nuns. If these two innings ere excluded the average of the remaining 38 innings is 48. His highest score was fa) 172 (©) 174 () 173 (a) 175 aa You have either reached a page that is unavailable for viewing or reached your viewing limit for this book. aa You have either reached a page that is unavailable for viewing or reached your viewing limit for this book. aa You have either reached a page that is unavailable for viewing or reached your viewing limit for this book. 616 Quantitative Aptitude ‘The mean marks of all the 25 students 1600 — ge -F -0m 46 (d). Income for 6th day in rupees = 15 «70-5 x 60-9 x 80 = 30 47 (C). The five even consecutive mumbers are 4,6, 8 10 and 12 44648410412 _ 40 Their average = STOPES B= 8 48 (d). Present age of § members (5 * 17 +3 x 5) years 100 years Present age of 5 members and a baby = 17 x 6 = 102 years ‘Age of the baby = (102 ~ 100) years = 2 years 19 (a). Sum of first nine numbers + sum of last nine numbers = 10.5 <9 +114 9=219 x9 = 197.1 Hence, the middle number 97.1 = 17 x 10.9 97.1 ~ 185.3 = 20 (c). To improve his average by 5 runs per innings hhe has to contribute 12 « § = 60 runs for the previous 12 innings. Thus, the average after the 13th innings = 96 - 60 = 36 24 (b). Average score before 17th innings 5-3 T= 34 Average score after 17th innings 43237 22 (a). Let the numbers be x, y and z. Then, 2 and = 3 ary t2= 9a % y or, y=30 23 (c). The sailor weighing 56 kg is replaced and the average is increased by 1 kg. Hence, the weight Of the new sailor is (56 + increase in toiat weight) 56 +188 56+ 8 = 64 kg S4T+144y 4 S474144y. seen 24 (c). Average of 5, 7, 14 and y Therefore, += 80% of 25 (c). 26 (0). 27 (0). 28 (b). =p =39 Present age of x = [(49 * 5} (4 «45 +4» 5)] years = 45 years Lot the rainfall on Wednesday be x em so that con the other 6 days, the total is also x, Since average rainfall for the week = 3 em x+x=3%7 or, x= 105 em Average monthly expenditure of 4 months = Rs 2,700 Total expenditure for 4 months = Rs 2,700 x 4 = Rs 11,000 ‘Average monthly expenditure of 3 months = Rs 2,940 Total expenditure of 3 months = Rs 2,940 * 5 = Rs 8,820 i) Average monthly expenditure of $ months = Rs 3,130 Total expenditure of 5 months = Rs 31,020 = Rs 1,560 Ci) ‘Total expenditure in the whole year = Rs 11,000 + 8,820 + Rs 15.650 = Rs 35,470. Saving during the whole year = Rs 5,330 Total income of the family during the year = Rs 35,470 + Rs 5,330 = Rs 40,800 Average monthly income during the year = fone “2 Let the average age of 8 men be x years Sum of the ages of 8 men = 84 years Now, according tothe condition of the question, average age of (6 men + 2 women) = (x + 2) years ‘Sum of the ages of (6 men + 2 women) A(x +2) = Re + 16 years Hence, itis clear that on replacing two men by two women, sum of their ages increased by 16 years Therefore, sum of the ages of two women = 20 + 24) + 16 = 60 years 60, 7B years Rs 3,00 Average age of the wome! aa You have either reached a page that is unavailable for viewing or reached your viewing limit for this book. aa You have either reached a page that is unavailable for viewing or reached your viewing limit for this book. aa You have either reached a page that is unavailable for viewing or reached your viewing limit for this book. aa You have either reached a page that is unavailable for viewing or reached your viewing limit for this book. aa You have either reached a page that is unavailable for viewing or reached your viewing limit for this book. aa You have either reached a page that is unavailable for viewing or reached your viewing limit for this book. aa You have either reached a page that is unavailable for viewing or reached your viewing limit for this book. aa You have either reached a page that is unavailable for viewing or reached your viewing limit for this book. aa You have either reached a page that is unavailable for viewing or reached your viewing limit for this book. aa You have either reached a page that is unavailable for viewing or reached your viewing limit for this book. aa You have either reached a page that is unavailable for viewing or reached your viewing limit for this book. 72 © Quantitative Aptitude and we write a: bs: 6: d. This is read as “a is 0 Bas cistod’. F since > = & te 32425628 ‘or exemple, since 5 = 5. we write 3:42:62 and say 3, 4, 6 and & are in proportion, Each term of the ratio $a 1a, b,c and d are respectively the frst, second, third and fourth proportionals. Here, a d are known as extremes and b, © axe known as means. Ru Two quantities ‘a and ‘bare said to be varying with each other if there exists some relationship between ‘a° and ‘b". 4 is called a proportional. Direct Proportion (Variation) ‘Two quamtities ‘a’ and ‘8° ate said to be directly Proportional if the increase (or decrease) in one quantity produces the increase (or decrease) in the other quantity by the same proportion. In other words, the ratio of ‘a’ and ‘" is a constant, ‘The statement “6° varies as ‘a’ is symbolically written as ba, If bce a, then b= ka, where & is a non-zero constant called the constant of variation. Inverse Proportion (Variation) ‘Two quantities ‘a" and ‘b* are said to vary inversely if the increase (or decrease) in one quantity produces the decrease (or increase) in the other quantity by the same proportion. ‘The statement vaties inversely as ‘a’ js symbolically 1 writtenas bee =, toa 4, mend = £ ie. ah=&, where isa non a a zeto constant called the constant of variation. ‘SOME BASIC FORMULAE Fomnula 1. If four quantities are in proportion, then Product of Means = Prosluct of Extremes For example, in the proportion a :B i: 0:4, we have be ~ ad. Product of means Product of extremes From this relation we see that if any three of the four quantities are given, the fourth can be determined. Formula 2. Fourth proportional Ifa: 6::¢:x,xisealied the fourth proportional of a, b, bxe @ We have, © > b ‘Thus, fourth proportional of a, 6, ¢ is a. ¢ Sor, r IMlustration 1. Find a fourth proportional to the numbers 2,5, 4. Solution Lot x be the fourth proportional, then 2:5 4:xore 5 ped 2 Formula 3. Third Proportional Ifa the third proportional of a, 6. a_t a We have, =o ‘Thus, third proportional of a, b is 2 Mlustration 2 Find a third proportional to the numbers, 25,15. Solution Let x be the third proportional, then 25 15 sors— 2521S 2215 ca LSxL5 25 x = os Formula 4. Mean Proportional Ifa:.x:::b,xis called the mean or second proportional of a, b. We have, 2 = or? = ab or x = Yah, Mean proportional of @ and 6 is ab. We also say that a, x, b ate in continued proportion Ulustration 3 Find the mean proportional between 48 and 12, Solution Let x be the mean proportional. Then, (invertendo) nis ie (Alternendo) aa You have either reached a page that is unavailable for viewing or reached your viewing limit for this book. aa You have either reached a page that is unavailable for viewing or reached your viewing limit for this book. aa You have either reached a page that is unavailable for viewing or reached your viewing limit for this book. 76 \ Quanttauve Aptitude SC 6. There are four numbers a, b, ¢ and d. (a) The number that should be subtracted from cach of these numbers so that the remaining numbers may be proportional is given by ad-be (xd) ~ (bee) Explanation Let x be subtracted from each of the number ‘The remainders are a —x, 6 —x, ¢ — x and d . Given: S— = 3 (a-N(d-x)=(b-Hle-9 = ad x(a +d) +7 = be (b+ 0) +8 and, their expenditures are given by (ba) dS(b-a) Rs adhe ad—be Explanation Let their incomes be Rs ak and Rs bl, respectively. Since each person saves Rs S, expenditure of first person = Rs (ak ~ 5) and expenditure of second person = Rs (bk ~ 5S) bk-S = akd ~ Sd = bke ~ Se +e (a+ ae = be do ahr en (acres becca = Mad — be) = (d- 08 or, k= SE tema ad~be adhe Bedside Therefore, the incomes of two persons are ad=o8 ,, Hd SC 6. (6) The number that should be added to each of ad be ad=be those numbers s0 that the new numbers may be proportional is given by bem ad G4a)- 070) Explanation Let x be added to cach of the numbers. ‘The new numbers are a +x, b+ x, 6+. and d +x, atx ote bay dex = (a+ a)(d +x) =(b+ lex) SB ad+xat dtr =berxbtote = (at dys —(b eye = be ~ ad bem ad G+d)- O40 Given: istration 14 Find the number subtracted from each of the numbers $4, 71, 75 and 99 leaves the remainders, which are proportional. Solution We have, a = $4, The required number = —2¢=e__ (atd)-(bte) 499 G44 99)— C1475) 7x7: | SC 7. The incomes of two persons are in the ratio of a band their expenditures are in the ratio of ¢ sd. If the saving of each person be Rs S, then their incomes are given by and their expenditures are ak~S and bk—S that is, ad —be id ad—be Mustration 15 Annual income of 4 and B is in the ratio of $: 4 and their annual expenses bear a ratio of 4:3. Ifeach of them saves Rs 500 a! the end of the year, then find their annual income. Solution We have, a: b= 5: 4,e:d=4:3 and S= 500. aStd 0) Annual income of 4 = 5x500x (3-4) (Gx3—4x4) = Rs 2500 bStd-e) ad be 4x500%«(3-4) (Sx3-4x4) = Rs 2,000 and annual income of & Wlustration 16 The incomes of Mohan and Sohan are in the ratio 7 : 2 and their expenditures are in the ratio 4: 1. If each saves Rs 1,000, find their expenditures. Solution We have, a : S= 1000, b=T:%e:d=4:1 and aa You have either reached a page that is unavailable for viewing or reached your viewing limit for this book. aa You have either reached a page that is unavailable for viewing or reached your viewing limit for this book. aa You have either reached a page that is unavailable for viewing or reached your viewing limit for this book. 730 21. 28, 29. 30. a. 32. 33. 3M 35. 36. a. © Quantitative Apeude ‘Two numbers are in the ratio 3 : 5. If each number is increased by 10, the ratio becomes 5 : 7. The numbers are (@ 15, 25 (©) 48, 60 (b) 30, 45 (@) None of these The ratio between two numbers is 2 : 3. If exch umber is increased by 4, the ratio becomes 5 : 7. The numbers are @) & 16 (b) 16, 24 (©) 24, 32 (@) None of these ‘Two numbers are in the ratio of S : 6. If $ is subtracted from each number, the ratio becomes 4:5. The numbers are (a) 23, 30 (©) 15, 20 ‘The ratio of present ages of Suresh and Mahesh is 7S. If after 6 years their ages will be in the ratio of 4 : 3, the present age of Mahesh is (a) 32 years (b) 36 years (©) 30 years (6) None of these (b) 30, 48 (4) None of these ‘The ratio of present ages of Sita and Gita is 4 : 3 If, 4 years before, the ratio of their ages was 2:1 the present age of Sita is (a) 8 years (b) 10 years (e) 12 years (d) None of these ‘Two numbers are in the ratio of $: 8, If 12 be added to each, they are in the ratio of 3 : 4. Find the sum of two numbers. fa) 43 (b) 39 47 (© None of these Two numbers are in the catie of $ : 7. If 25 be subtracted from each, they are in the ratio of S : 59. Find the difference of the two numbers, (@ 48 (b) 52 © 24 (@) None of these When x is added to each term of 7 becomes 2 : 3. The value of x is @7 (b) 1 ©5 (G) None of these Find the number which, when subtracted ftom the terms of the ratio 12: 17 makes it equal to the 13 the ratio ratio 2: 3, @) 2 (b) 6 (8 (W) None of these ‘The value of & that must be added to 7, 16, 43, 79 so that they are in proportion, is @) 7 (bs © 9 (a) None of these What should be subtracted from 15, 28, 20 and 38 so that the remaining numbers may be proportional? @2 (b) 4 © 6 (@) None of these 38. 2”. 4. al, om a. 44. 46. ‘The number that must be added to each of the mumbers 8,21, 13 and 31 tomake the ratio of first two numbers equal to the ratio of last two numbers is (a) 5 (0) 7 i) 9 (@) None of these The incomes of 4 and B are in the ratio 3: 2 and their expenditures in the ratio 5 : 3. If each saves Rs 1000, 4°s income is (@) Rs 5.000 (b) Rs 6,000 ) Rs 8,000 (@ None of these The annual incomes and expenditures of man and his wife are in the ratios of 5: 3 and 3 > 1, respectively. If they decide to save equally and find a balance of Rs 4,000 at the end of year, their incomes were (a) Rs 5,000, Rs 3,000 (b) Rs 6,000, Rs 4,000 (c) 883,000, Rs 2,000 (4) None of these The incomes of Gupta and Verma are in the ratio 9: 4and their expenditures are in the ratio 7: 3. If each saves Rs 2,000, then Gupia’s expenditure is (a) Rs 60,000 (6) Rs 80,000 (©) Rs 70,000 (@) None of these Ina mixture of 60 litres, the ratio of milk and water is 2: 1. What amount of water must be added to make the ratio of milk and water as I : 2? a) 75 litres () 55 litres (c) 60 litres (8) None of these A mixture contains alcohol and water in the ratio of 12: 5, On adding 14 litres of water, the ratio of alcohol to water becomes 4: 3. The quantity of alcohol in the mixture is (a) 18 titres () 24 litres (©) 26 lites (2) None of these If an alloy contains copper and silver in the ratio 3:7, then the percentage quantity of silver in the alloy is (a) 90% (b) 70% (c) 60% (a) None of these Two alleys contain zine and copper in the ratio of 2: Land 4 : 1. In what ratio the two alloys should be added together to get as new alloy having zine and copper in the ratio of 3 : 1? (7:5 (b) 5:7 (oy 3:8 (a) None of these Mixture of milk and water has been kept in two separate containers. Ratio of milk to water in one of the containers is 5: 1 and that in the other container is 7 : 2. In what ratio the mixtures of these wo containers should be added together so that the quantity of milk in the new mixture may become 80 per cent? (a) 2:3 lo 45 (by 3:2 (d) None of these aa You have either reached a page that is unavailable for viewing or reached your viewing limit for this book. aa You have either reached a page that is unavailable for viewing or reached your viewing limit for this book. aa You have either reached a page that is unavailable for viewing or reached your viewing limit for this book. 74 26. 27. 28. 29, 30. ai. 1 Quanttaive Aptitude @) 9% (bo) 144 (©) 114 (141 Rs 770 have been divided among 4, B and C such that A receives 2/9th of what B and C together receive. Then 4's share is (@) Rs 140 (b) Rs 154 © Rs 165 (@ Rs170 What least number must be subtracted from each of the numbers 14, 17,34 and 42 so that the remainders are proportional? @) 0 © 2 1 @7 xand y vary inversely with each other. When 2 is 12, 1) is 9. The pair which is not a possible pair of corresponding values of x and y is (@) 9 and 12 (b) 18 and 6 (©) 24 and 18 (a) 36 and 3 ‘The difference between a two digit number and the number obtained by interchanging the digits is 36. ‘What is the difference between the sum and the difference of the digits of the aumber if the ratio between the digits of the number is 1 : 2? @ 16 (be) 4 8 (d) None of these At Narmada Sarovar Bachao demonstration, supporters of Ms Patkar outnumbered the police by 9: 1. The police arrested 135 NSB supporters averaging 5 for every 3 policemen. How many supporters of NSB were there in the demonstration? (a) 405 (b) 665 (©) 1215 (@) None of the above Monthly incomes of two persons are in the ratio 5:4 and their monthly expenditures are in the ratio of 9: 7. If cach person saves Rs 500 per month, then what are their monthly incomes? (@ Rs 8,000 and Rs 10,000 @) Rs 3,750 and Rs 3,000 (©) Rs 5,000 and Rs 4,000 (None of these Which of the following is the ratio between a number and the number obtained by adding one-fifth of that number to @ 6:5 (0) 5:6 (5:4 4:5 ‘The intensity of illumination on a surface from a source of light varies inversely as the square of the distance of the surface from the source. The effect ‘of moving a piece of paper 3 times as far from the source is to (a) Divide the intensity by 3 32. 33, 35, 36. (®) Multiply the intensity by 3 (©) Divide the intensity by 9 @ Multiply the intensity by 9 Suppose » varies 2s the sum of two quantities of which one varies directly, as x and the other varies inversely as x. If y = 6 when x = 4 and y= 34 when x= 3, then the relation between x and vis @ yextt ) year+ © yo2+8 @y- 2-8 x x The ratio of the age of a man and his wife is 4:3. After 4 years this ratio will be 9:7. If atthe time of the marriage, the ratio was 5 : 3, then how rmany years ago they were married ? (@) 12 years () 8 years (©) 10 years (@ 15 yours (©) None of the above Pressure varies inversely with volume while temperature varies directly with volume. At a time, ‘Volume ~ 50m’, Temperature ~ 25° K and Pressure~ I atmosphere. Ifthe volume isincreased to 200 m?, then the temperature will be (@) 100° K (b) 50° K © pK @ 0K (©) None of these For cach 200 rupees spent by the research department, sales department spends 20 rupess. For every 400 rupees spent by the sales depertment, the advertising department spends 150 rupees. The triple ratio of the money spent by the research department to the money spent by the sales department to the money spent by the advertising department can be expressed as (a) 40823 (© 20:4:01 (b) 80:83 @ 2:1:5 2 varies directly as y’ and when x = 6, y= 3. Which of the following equations correetly represents the relationship between x and y? &) 37 = 6? (a) 3 =4y* @ 6 =39 (@) 3 =2y° tis given that y« J: Hx= 1, then the value of y will be @1 m0 @-1 (@ None of these aa You have either reached a page that is unavailable for viewing or reached your viewing limit for this book. aa You have either reached a page that is unavailable for viewing or reached your viewing limit for this book. aa You have either reached a page that is unavailable for viewing or reached your viewing limit for this book. 718 84, © Quanitaire Aptitude ‘The sum of three numbers is 98. If the ratio of the first to the second is 2 : 3 and that of the second to the third is 5 : 8, then the second number is (a) 20 (6) 30 (©) 38 @ 8 ‘Two numbers are such as the square of one is 224 less than & times the square of the other. If the number be in the ratio of 3 : 4, their values are ANSWERS Exercise 7.1 Lo 2 © 2 @ & @ 2% © ® mo m @ nm @ 18 » |) mm © % @ % 6) mB mh O) 2 © MW) 3 W % & 1 © &% © & & 4 Exercise 7.2 Lo 2m 2 @ £ Oo Oo m™ @ © 2 © mn © ® @ BO mw ©) Exerci Que: L@ %@@ 2 & & oo) © @ ww Bw @ n@® Bo Bw m @ BO BO moO ® & Bm * @ RH m% @ 16 2 @ 8 w® Hw @ © o 2@ <8 8% @ HO) 6 ©) 66. o @ & © Bn O@O &@ Bw % © CC ae a 86. @ 12,16 (©) 912 (0) 6,8 (@) None of these If varies as QP and the three corresponding values of P, G, R be 6, 9, 10 respectively, then the value of P when Q = 5 and R= 3 is @2 () 3 ©! (@) None of these Level T o 6 © 7 @ & © mB Oo MH © & & 6 @& 2 @) 2 3), 2» @ 3 ©) 3 @ 32 & 37. @ 8) YD) BA) Level If B® 6 @ 7% @® & ® B © kh © 1S He @ 2 © Wm ©) B © wm WO ‘ions. from Competitive Examinations © 609 27% & © B © MH @ 6% © 16 © uno 2 oO Bw wm wm & 2» Oo Mw FO 3 & 7 & O % @ MM 45. 0) 46) a OL © 6h oe OE 6% © % @ Mm @ Bm oO Th 0) Tb) | 8 @ 8.) 8H) aa You have either reached a page that is unavailable for viewing or reached your viewing limit for this book. aa You have either reached a page that is unavailable for viewing or reached your viewing limit for this book. aa You have either reached a page that is unavailable for viewing or reached your viewing limit for this book. 722 ® Quamsitauve Aptitude 44 (b), Here, a= 3 and b = percentage quantity of silver in the alloy b 7 2) so = (2) 190% = 10% (a) = 00% = (ep) sta 45(0h Wehwe,a:b=2:1,e:d= 401 Let the two alloys be mixed in the ratio x : Then, percentage quantity of zine in the new alloy alo yD =| 2th erd | «100% xy 2x, 4y =|3—| « 100% xy lOx+12y — 10e4 129 9005 1 iS(rry) a. Since the ratio of zine and copper in the new allay is. 3: 1 percentage quantity of zinc in the new alloy 3 100% = 300% of, 87% (2) el Exercise.’7.2 1 (b). Ratio of speed = Ratio of distance. Therefore, ratio of distance =4: 3. Hence, the man travels 36 miles and the woman 27 miles Let, the price of branded PC be = 64x and that of assembled PC= 27x, Let, after years the prices of both will become equal Then, 0,9)" 6x =, SA 27 (3 Henee, n= 3 years The box to paper ratio is 3 : 22. To arrives the weight of the box, we firstdivide 36000 by 25 to get 1440, Multiplying this by 3 we get 4320 which is the weight of the box. Therefore, weight of the paper is 36000-4320 =31,680 kg, 2(b). 1.2)" 21 3 (a). 4(C). Given: 44 =SB= 7C oe Be > 35 "28 So, 4:8: 735:28:20 46 (a). 5(d). 6 (d). From (1) and (2), we get MDW 8 oe aoe any 045849) ze a or, S¥=3y Onn zy 5 Hence, the alloys should be mixed in theratio 3: § Here, a: 65 a 7:2 Lot the mixtures of the two containers be added together in the ratio of x: y. percentage quantity of milk in the new mixture Se, =| 245244 | 00%, i “ 100% = 554422 1099, (ry) Since the percentage quantity of milk in the new mixture is 80% A5e442y 4x7) (45x +429) x 5=4 x 54x +») 225x + 210y = 216x + 216y 9x~ Gy orxiy 233 «100% = 80% buy Level IL 35 ene (35+ 28+ 20) . 581-245, dCs ot S81 = 140 and Csring = 20 x s8-= Thus, difference between 4°s score and B's score =(245~ 140) Since the ratio is 20 : |, the number of boys can be 20,40, 69, ..and numberof girls ean be 1, 2,3. ne ‘Thus, as the number of students is not krown, we cannot say for sure the number of girls to be required Hence, datais insufficient Letthe price of B per kg be Rs X. Then the price of Aperkg= Rs3 Thgof Ccontains 2/7 kg of 4 and 5:7 keof B Price of | kg of C= (27) x 34+(S7)¥=(1).X By the given condition, 11/7 =5.20-0.80 =Rs4.40 => X=4.40% (711) =Rs2.80 Hence the price of B per kg= Rs 2.80 aa You have either reached a page that is unavailable for viewing or reached your viewing limit for this book. aa You have either reached a page that is unavailable for viewing or reached your viewing limit for this book. aa You have either reached a page that is unavailable for viewing or reached your viewing limit for this book. 76 23 (b). Let the number of one rupee coins, 50 paise B Quintiaive Aptinde 2 SI-9E=500 i) I~ TE = 500 => 1 1000,£= 500 Monthly incomes of the two persons are Rs 5,000 and Rs 4,000, Also, 30 (b). Let the number be x Only option (¢) satisfies the equation (iii) Required ratio = — coins and 25 paise coins respectively be 2k, 3k and 4k. 34 (ce). Fa 2k + 3k x (0.50) + 4k = (0.25) = 216 § > k+ 5k + k= 216 92 (d). Let y «p+ q, where pw xandg = + = 45k =216 = k= 48 - : 2 y= K ta No. of 50 paise coins = 3k = 144 N f 24 (a). A= 28+) po ieg= Xo y= K[ Mer X 94 10 4 N) +B+c-1 34+ = ~ 18 whens 3 (3a S1A=71002 4-140 MK = 2 and NK l4-x Maw . 2x SS lunigbeult ic P 4 and so y= 2x-% OO) og tase x Kx x => 588 - S6x +x? = 578 Skt? 33 (a). Man’s age = 4k, say = x=? Wife's age = 3k, say 1 k _ ake 9 28 (0). sa me x= 2 Giprpaecs : Man's age ~ 32 years where é is constant. When x= 12, p Wife's age = 24 years. a Eimiagcans Beasidindt a, Homie Suppose they were married x years ago Wea, ee me ape « BEE sSayey we 432ek wox 3 ‘Therefore, x= 24 and y = 18 is not possible 34 (a). +> Temperature « Volume 27 (e). Let two digit number be 100 +b it reapeneiies as": Given: (10a + 6) - (106 + a) = 36 ‘Volume = $0 m’ Be ODP ea: IfVolume = 200m’, then an bad = bm 4 ‘Temerpature = 100° K Given, $= > bora 35 (b). RD: SD: AD= 80: 8:3 21s a=4andb=8 (ie., 4000 : 400 : 150) 28 (d). Number of policemen 29 (c). Let the monthly incomes of two persons be S/ 36 (d). Required number = (8 + 4) - (8 - 4) 135 = 81 3 Since number of supporters: number of policemen = 9:1 therefore mumber of supporters must be ns 37 (a). and 4/ and their monthly expenditures be 9 and ‘TE respectively. aa You have either reached a page that is unavailable for viewing or reached your viewing limit for this book. aa You have either reached a page that is unavailable for viewing or reached your viewing limit for this book. aa You have either reached a page that is unavailable for viewing or reached your viewing limit for this book. 7H 8 Quintiative Aptiude Quantity oftin in 1 unit ofalloy = 79 (c). Let the numbers be Sx and 4x Given 5x-4r—10 = x= 10 so Larger number = 10 80 (a), We can make use of the options to solve this problem (a) 7* 480-2 * 130 = 3620 (b) 7 * 480 +3 « 130= 3750 (©) 2* 480 +7 « 130 = 1870 Hence option (a) is the corect option A 3KS NS B 2 SRA BUC) 4x3 7 20°C” oxd 8 B 2x7 140, © _ 7x24 168 C7 taxt “168! D 5x24 ~ 120 A, 1Sx7 B 20x7 A Now 4 = “3B A:B:C:D Hence, C gets the maximum share 82 (b). 435+ 217 = 535 By approximations 5 = 10, T= 5 43.104 21-5 = 535 ints t0 tie = 10 : 5 =i 84 (b). 83 (e). Let x pairs of brown socks were ondered, Let P be the price of a brown pair ‘Therefore price of black pair of sock = 2P Then, 4P = 2Py= 1.5 GP + 8P) BxP + 24P, uuu A:B:C= 10:15:24 Let the numbers be 105, 15x and 24x, then lox + 152 + 2a = 98 = Wxr=98 = x second number = 15x = 15 x 2= 30 85 (b). Let the numbers be 3x and 4x, then Lax? = 8 * (9x)? = 224 = eet 72x? 224 = s6e=224 = Values of number 86 (c). Pa OR, P= 6, 0=9,R=10 = P=K QR, where & is the constant of proportionality = 6=Kx9x10 > K= 15 = SP=OR. When Q = 5 and & = 3, we get P= 1 EXERCISES FOR SELF-PRACTICE Exercise 7.4 Rs 675 was divided between 4, B and C. If each of them had received Rs 5 less, their shares would have deen in the ratio of 1: 2: 3. How much did B receive? (@ Rs 11250 (b) R825 (©) R820 (@) Rs 225 A merchant blends two varieties of tea from two different gardens, one costing Rs 45 per kg and ‘ther Rs 60 per kg, in the ratio of 7 : 3, respectively. He sells the blended variety at Rs 54.45 per kg. The profit per cent in the transaction is @ 3% (b) 10% 1, J, © 97% (@) Ny Level I ‘The ages of Amit and Puneet are in the ratio 2:3, After 12 years, their ages will be in the ratio, 11 : 15. The age of Puneet is (a) 32 years (b) 42 years fe) 48 years (a) 56 years, ‘The proportion of zine and copper in a brass piece is 13: 7. How much zine will be there in 100 kg, of such a piece? (@) 20kg (ty 33kg te) SS kg () 65 ke 15 litres of mixture contains 20 per cent alcohol and the rest water, If 3 litres of water be mixed in it, the percentage of alcohol in the new mixture will be aa You have either reached a page that is unavailable for viewing or reached your viewing limit for this book. aa You have either reached a page that is unavailable for viewing or reached your viewing limit for this book. aa You have either reached a page that is unavailable for viewing or reached your viewing limit for this book. 82% Quanitative Aptitude Formule. (a) IFthe capitals of two partners be Rs C, and Rs C; forthe periods 4 and f, respectively and the toral profit be Rs P, then shares of the partners in the profits are: 5 xP. xe Rs | GA? | ond Rs | GX2*P Gat Gn Git Gh (b) Ifthe capitals of three partners be Rs C,, Rs C and Rs C; for the periods ff and fy, respectively and the total profit be Rs P, then shares ofthe partners in the profits are -( xh xP jis xix? Gat Gn tGn) (Gat Gn tn mape| oP GhtGn+Gy llustration 3 There are three partners 4, B and C in a certain business. 4 puts in Rs 2,000 for 5 months, B Rs 1,200 for 6 months and CRs 2,500 for 3 months. Find the ratio of their shares in the profit. Solution Here, c; » 4, =2000 x = 7200 and Cy * fy = 2500 x3 1. Profit of 4 : Profit of B : Profit of C FOXX = 10000 : 7200 : 7500 or 100 : 72 : 75 1200 SC 2. if the capitals of three partners are invested in the ratio C, :C, Cy and their profits are in the ratio P,: Py: Py then the ratio of timing of their ABB investments = ok Ga G Mlustration 2 4, B are two partners in a business. A contributes Rs 1200 for 5 months and B Rs 750 for 4 months. Iftota profitis Rs 450, find their respective shares. Solution We have, C, = 1200, C; = 750. f, 4 and P = 450. Profit share of A Gx eP __1200*5%450 Gh+Gt 120054 750%4 2700000 =Rs300 9000) and, profit share of B 750 4* 450 GREP 1200*5+ 750% 4 Gh+Gn 1350000 = Rs 150 9000 ‘SOME USEFUL SHORT-CUT METHODS SC 1. (@) IF the capitals of two partners be Rs C, and Rs C, for the periods f, nd t, respectively, then Profitof 4 _ Gx, Profitof BC *ty () If the capitals of three partners be Rs Cj, Rs C, and Rs C, for the periods ff, and f, respectively, then profit of 4 : profit of B: profit of C=C, * fy 2 Cy ¥ ty: Cy ty Note: If there is a loss in the business, then Loss of 4: Loss of B : Loss of HCG GK Mlustration 4 Anu, Manu and Tanu invested capitals in the ratio 4 : 6: 9. At the end of the business term, they received the profits in the ratio 2 : 3 : 5, Find the ratio of time for which they invested their capitals. Soluition we have, C+ C; + C, and, Py: Pai Py Therefore, the ratio of time for which Anu, Manu and Tanu invested their capitals or, 9:92:10 SC 3. Three partners invested their capitals in a business, If the timing of their investments is in the ratio f, ff, and their profits are in the ratio P, : Py: Py, then the ratio of their capitals invested is RB 4 at Mlustration 5 Gupta, Singhel and Kansal start a ‘business. Ifthe ratio of their periods of investments are 1:2: $ and their profits are in the ratio of 3 :4 : 5, find the ratio of capitals of Gupta, Singhal and Kansal. Solution We have, Pp: Py: Py= 3:4: 5 and tigihal:2:5 + Required ratio = 4,8, he 45 == of 3 125 ‘Thus, Gupta, Singhal and Kansal invested their capitals aa You have either reached a page that is unavailable for viewing or reached your viewing limit for this book. aa You have either reached a page that is unavailable for viewing or reached your viewing limit for this book. aa You have either reached a page that is unavailable for viewing or reached your viewing limit for this book. 86 Exercise 8.3 Ea 4 © Quantiative Aptitude 4, B, C subscribe Rs 50,000 for business. A sub- seribes 4,000 more than 8 and B Rs 5,000 more than C. Out of total profit of Rs 35,000 A receives (@) Rs 11,900 (b) Rs 8,400 (©) Rs 14,700 (@) Rs 13,600 4 and B enter into a partnership with Rs 50,000 and Rs 60,000 respectively. C joins them afterx months contributing Rs 70,000 and B leaves x months before the end of the year. If they share the profit in the ratio of 20 : 18 21, then find the value of x. 9 (b) 3 ©) 6 @s Rs 120 are divided among X, Y and Z so that Js share is Rs 20 more than ¥°s share and Rs 20 less than Z's share. What is Y's share? (a) Rs 40 (b) Rs 30 (©) Rs25 (@) Rs 20 ‘Surendra, Rajendra and Manindta invested some amount in a business in the ratio of $ : 7: 6 respectively. In the next year, they increased their investments by 26%, 20% and 15% respectively. The profit earned during the second year should be jisributed in what ratio among Surendra, Rajendra ‘and Manindra? () 31:27:21 (©) 26:20:15 (by 21:28:23 (@) Cannot be determined X and Y start a business. X invests Rs 3,000 for 4 months and ¥ invests Rs 2,000 for 6 months. How much should X’be paid out of a total profit of Rs $00? (a) Rs 200 (b) Rs 300 (©) Rs250 (a) Rs 350 M,P and Q jointly thought of engaging themselves in a business venture. It was agiced that M would invest Rs 6,500 for 6 months, P, Rs 8,400 for S months and Q, Rs 10,000 for 3 months. M was to be the working member of the group for which he was to receive 5% of the profits, The profit earned was Rs 7,030. Calculate the share of P in the profit, (@) Rs2,660 (b) Rs 1,900 (©) 82,840 (@) Rs 2,800 Atul and Babita enter into a business partnership in which Atul contributes Rs 2,000 for 9 months and Babita contributes Rs 5,000 for 7 months. 10. nD. 1B. Questions from Competitive Examinations A profit of Rs 1,100 will be divided between Atul and Bebita in the ratio of @ 6:1 © 7:9 (b) 2:5 (@) 18:35 P,Q and R enter into a partnership with investments Of Rs 3,500, Rs 4,500 and Rs 5,500. In the first six months, profit is Rs 405. What is P’s share in the profit? (@) Rs 200 (©) Rs2s0 (b) Rs 105 (@) Rs AST An amount of money is to be distributed among 4, Band C in the ratio 3 : 1: 5. The difference between 's and C’s share is Rs 3,600. What is the total of A's and B’s share? (@) Rs5,400 (b) Rs 3,600 (©) Rs2,700 (@) Rs 1,800 A and B invested in a trade. They eamed some profit which they divided in the ratio 2 : 3 of their investment. If 4 invested Rs 40 then the amount invested by B is @ Rs30 () Rs 60 © Rs (@) Rs 100 A started a business with Rs 4,500 and another person B joined after some period with Rs 3,000. Determine this period before B joined the business if the profit at the end of the year is divided in the ratio 2:1 (@) After 3 months (b) After 4 months (©) After 6 months (@) After 24 months ‘A began a business with Rs 4,500 and was joined afterwards by B with Rs 5,400. If the profits at the end of the year were divided in the ratio 2: 1, 8 Joined the business after (a) 5 months (©) 4 months (6) 6 months (a) 7 months ‘Three partners altogether invested Rs 1,14,000 ina business. At the end of the year, one got Rs 337.50, the second Rs 1,125.00 and the third Rs 675 as profit. What is the percentage of profit? (a) 48% (b) 3.8% (©) 18% (@ 28% aa You have either reached a page that is unavailable for viewing or reached your viewing limit for this book. aa You have either reached a page that is unavailable for viewing or reached your viewing limit for this book. aa You have either reached a page that is unavailable for viewing or reached your viewing limit for this book. a0 Quantitaive Aptiude Exercise 8.2 4a). 20). 3c). 4(b). 5 (a). Let Tr and Sv be the investments of 4 and 8 respectively Let 3y and be the loss of and profit of B respectively then, 7x~ 33+ Sx# y~ 6000 = 6x—y =3000 i Sx+5y =9500 > xt y=1900 i) ()+Q)>¥=700 Hence, total money invested ~ 2x + 5x Rs8,400 Rs 300 s tobe divided among 4, B and € Let 4 be d’s share, B be B's share and Che C's, share and A Be BiC A+C ASB 1+B+C 1 WAtB+C) 2 4 s 100 Initial Capital = Rs 2,100. 4°s capital continues for 12 months. £8°s capital continues (12 —4) = ¥ months. Let puts in Rs. => 210% 12=1 x 891= 2100 128=Rs3,180 Let the total profit be Rs. Amount of prof Ratio of capitals = $000 : 6000: 4000 or l(a} i] “() A's share ax ~ ory =3000 is ‘Thus, the total profit is Rs 3,000 Feo ea 2250010 Interest paid by B= Re| ps )-m220 5c) 7d). 8 a). 9(c). 410(¢). 11(b). Level II Teal mony raved by 4 ns =HE saan) 230) ‘Total money recevied by # ~1440 =| u =a( 208) Jstss0-250] Given: 343060 43060) Hence, the total profit = or or= x=9180 Rs 9,180 A'S profit + 1200 = 3900 = 4°s profit=Rs 2,700 B's profit ~ (80000/60000) * 2.700 ~ (4/3) 2,700 = Rs 3,600 Let they invest money for x, y and z months then Sv:6p:8:=5:3:1 I 8 Ratio of investments of 4, B and C = 1200024: 16000 24: 1500016 = 18:24: 1S : @4el Let the initial investments of A, Band C be 3x and 7x respectively. nvestmeat afier one year pr 45600, Sx, 7+ 337600 Given 3x~45600 : Sx: T+ 337600 $24:59: 167 of x = Rs 47,200, therefore, investment of 4 = 3» 47200=Rs 1,41,600 Sie people contribute a total af Rs 180. Let the seventh person contributes Rs x Eighth person contributes Rs 55 ‘Total contributions of these eight persons = 235 +x 235 4x Now x a +10 7 = 1-hass)+ 100-45 Zx= 30235) Total collection 235 +48 = Rs 280 Rate in which the rentis to be divided =: 8 : 12 Ait sshtctrn= S600 R300 aa You have either reached a page that is unavailable for viewing or reached your viewing limit for this book. aa You have either reached a page that is unavailable for viewing or reached your viewing limit for this book. aa You have either reached a page that is unavailable for viewing or reached your viewing limit for this book. B14 1 Quantitative Aptitude 6 (a) Rs 5,800 (b) Rs 6.800 (©) Rs7,800 (a) Rs 8800 A and B are partners in a business of which they share the profits as follows: B fist receives Rs 10,000 together with S per cent fo the balance for manzging the business; the remaining profit is divided between A and B in the proportion of 5: 4. If B's income from the business after deducting income tax at 40%, is Rs 16,200, find 4’s gross income. (a) Rs 50,000 (b) Rs $5,000 (©) Rs$6,000 (@) Rs 54.000 Directions (Questions 7 and 8): Refer to the following inf Th Rs of formation to answer the questions. ee partners 4, B and C poo! in Rs 3,000, Rs 4,000 and 5,000 respectively in business. 4 receives 15 per cent the profits as Manager and B receives 10 per cent of the profits as Assistant Manager, after which the remaining profit is divided in proportion to the capital subseribed by each. If 4 receives Rs 540, then 7. The share of B will be (@) Rs 540 (b) Rs 560 (6) Rs $00 (a) Rs 600 8. The share of C will be (a) Rs540 (b) Rs 560 (©) Rs 500 (@) Rs 480 9. A and B enter into a partnership. A pus is Rs 2,000 but at the end of 3 months, withdraws Rs 500 and again at the end of 8 months withdraws Rs 300. Out of ¢ total profit of Rs 900 at the end oF the year, B's share was Rs 400. Find 8's capital (a) S120 (b) Rs 1,130 (©) Rs 1.430 (@) None of these 10. 4, B, C start in business together; 4 puts in Rs 1,200,000, B Rs 1,440,000 and C’Rs 2,160,000. At the end of 3 months C leaves, and at the end of 7 months more B leaves, both taking out their capital with them, but the profits are not divided til the end ANSWERS Exercise 8.4 L@ 2@M 2 M 4 & ~ 69 © % % wo B @ mn @ B® 2% @ @ @ Exercise 8.5 L@ 2@ 2 @ 4 @ X69 © @® wo BO 12. 13. 14, 15, 13. 1B. of the year, when C receives Rs 12,000. How much will 4 and B together then receive? (a) Rs64,000 (b) Rs 48,000 fe) Rs24,000 (A) None of the above ‘Three people invested x,y, z amount of money ina project, Next year their share increases by 20 pet cent, 25 per cent and 12 per cent. Then what will be their profit sharing ratio next year? (a) 20x: 17Sy:1122—(b) 200r:225p:2125 (e) W20r: 125112 @) ROK: 75p: 18K Vand Ystarted a business with Rs 5,000 and Rs 6,000 respectively. After seven months, X withdraws Rs 1,000 and Yinvests Rs 1,000 more. At the endof the year, thoir profitsamounted to Rs 10,800. Find ¥'sshare inthe profit. (a) Rs4500 b) Rs 5,000 (©) Rs6.300 (€) Rs-4800 Pandey and Pradeep invested amounts in the ratio 4 : 5 in a business in which Pandey was the managing partner. They got profit of Rs 50,000 after some time, It Pancey’s salary is included, their total incomes are inthe ratio of I: 1. Find Pandzy’s salary fa) Rs5,000 (6) Rs 10000 Ce) RsR5,000 (4) Rs8,000 A and 8 started a business with some investment, 4 invested 36 per cent of the total investment and received 9/17th part ofthe total profit at theend ofthe ‘year. If A stayed in the business throughout the year, for how many months did B invest his amount? (e) 6 (07 © 8 @9 A,B, Cand D enter into a partnership. 4 invested Rs 5,000 for 8 months, # invested Rs 6,000 for 10 months, C invested Rs 3,750 for 8 months and D investd Rs 15,000 for 6 months. Find the combined share of 4 and D in the total profitof Rs 33,000. fa) Rs10.500 (b) Rs 13500 (©) Rs18,00 (d) Rs 19500 Level I © €@ 2% © & @ Oo & @ b& © Bm © Level II ® 60 2% ® & @ o mw @® 6B @ aa You have either reached a page that is unavailable for viewing or reached your viewing limit for this book. aa You have either reached a page that is unavailable for viewing or reached your viewing limit for this book. aa You have either reached a page that is unavailable for viewing or reached your viewing limit for this book. aa You have either reached a page that is unavailable for viewing or reached your viewing limit for this book. aa You have either reached a page that is unavailable for viewing or reached your viewing limit for this book. aa You have either reached a page that is unavailable for viewing or reached your viewing limit for this book. aa You have either reached a page that is unavailable for viewing or reached your viewing limit for this book. 98 8 10. ML, 22 1B. 14, © Quanttatwe Apainude Sardar Singh bought 200 dozen oranges at Rs 10 a dozen. He spent Rs $00 on transportation. He sold them at Re I cach. What was his profit or loss per cent? (a) 4% tb) 6% © 5% (d) None of these Mr Verma sold his scooter for Rs 10,500 at a gain of 8%, Find the cost price of the scooter. (a) Rs 10,300 (b) Rs 10,700 (©) Rs 10,000 (a) None of these “Suresh buys a camera for Rs 1800 and sells it at 10% loss. Find its selling price (a) Rs 1,620 (b) Rs 1,730 (0) Rs 1,650 (@) None of these Hemant purchased 120 reams of paper at Rs 80 per ream, He spent Rs 280 on transportation, paid octroi at the rate of 40 paise per ream and paid Rs 72 to the ccolie. If he wants to have a gain of 8%, the selling price per ream must be (a) Rs 89 {by Rs 90 fe) Rs 95 (d) None of these A shopkeeper loses 7% by selling a cricket ball for Rs 31, For how much should he sell the ball so as to gain 5%? (@) Rs 50 (b) Rs 65 (©) Rs 35 (d) None of these A shopkeeper sold some articles at Rs 35 perarticle and guined 40% on it. What would be the selling ce of cach article to get 60% profit? (e) Rs 40 (b) Rs 45 (©) Rs 50 id) None of these Aman bought apples at the rate of 6 for Rs 20 and sold them at 4 for Rs 16. His estimated profit per cent is (@ 23% ©) 20% A fruitseller buys 10 bananas for Rs 14 and sells 12 for Rs 15. Find his percentage gain or loss. 5 [b) 18% [d) None of these wt ) 103% toss (0) 9% gain (b) 10 5% eatin (d) None of these Hfeggs are bought 12 for Rs 10 and sold at 10 for Rs 12, What is the gain or loss %. (a) 40% loss (b) 44% gain (©) 44% loss [d) None of these Ifthe cost price of 21 watches is equal to the selling price of 18 of them, what will be the gain per cent, in this transaction? @ 6 (b) 7% (©) 165% (d) None of these 15, 16. te 18. 19, 20, 21, 2 2B. 24, A shopkeeper gains the cos! of 8 metres of thread by selling 40 metres of thread, Find his gain per cent (a) 19% (b) 20% (d) None of these the selling price of 2/3 rd ofa certain quantity of nnilk be equal to the cost price of whole milk, what will be the gain per cent in this transaction? (a) 50% (b) 48% (©) 53% (d) None of these A shopkeeper sells 20 pencils for the same money as he paid for 25, What is his gain per cent? (@) 20% (b) 2: (©) 24% (d) None of these Mohit lost 18% by selling a bicycle for Rs 1.230, ‘What per cent shall he gain or lose by selling it for Rs 1,600? (@ 3% Joss (b) 4% gain 2 (©) 65% gain (@) None of these A shopkeeper sells an article ata gain of 10%. Had he sold it at 2 loss of 20%, its selling price would have been Rs 180 less, What is the cost price of the seticle? (a) Rs 630 (©) Rs 580 (b) Rs 600 (d) None of these A person sells 36 oranges per rupee and suffers a loss of 4%, Find how many oranges per rupee to bbe sold to have a gain of 8%. (a) 1/32 tb) 5 ©) 16 (a) None of these A person sells a colour TV at 10% below cost price. Had he received Rs 1,494 more, he would 1 have made a profit of 125-%. What was the cost price of colour TV?” (@) Rs 6.400 (©) Rs 6.640 (b) Rs 7,200 () Note of these Vijay sold a watch at a gain of 5%, Had he sold it for Rs 72 more, be would have gained 13%, Find the cost price of the watch (@) Rs 900 (by Rs 910 (©) Rs 870 (4) None of these Sita sells a calculator to Gita at a gain of 17% and Gita sells it to Anu at a Joss of 25%, If Anu pays Rs 1,842.75 for it, what did Sita pay for it? (@) Rs 2.080 (b) Rs 2,100 (©) Rs 2.110 (d) None of these A buys an article and sells it to B at a profit of 10%, B sells it to C gaining 20 %. IfC gives Rs 924, what did 4 give? aa You have either reached a page that is unavailable for viewing or reached your viewing limit for this book. aa You have either reached a page that is unavailable for viewing or reached your viewing limit for this book. aa You have either reached a page that is unavailable for viewing or reached your viewing limit for this book. a2 n. Exercise 9.3 3. Quaneauve Apuide increased by 600%, Initielly the manufacturer was getting only 140% profit, What is the percentage increase af his profit? (a) 10% ©) 0% (b) 14% (d) cannot be determined The marked price of a wateh is Rs 1,600, The shopkeeper gives successive discount of 10%, 4% 10 the customer. If the customer pays Rs 1.224 for the ‘watch, find the value of r fa) 10% (c) 25% A trader sells goods to a customer at a profit of k% ‘over the cost price, besides it he cheats his customer by giving 880 g only instead of 1 kg. Thus his overall profit percentage is 25%. Find the value of & the then The cost price of 20 articles is the same selling price of x articles. If the profit is 25% the value of x is (a) 25 fc) 16 tb) 18 id) 15 A grocer bought 10 kg of apples for Rs 81 out of which one kg were found rotten, If he wishes 10 make a profit of 10%, then he should sell it at per kg, (a) Rs 9.00 (b) Rs 9.90 fe) Rs 10.10 fd) Rs 10.20 A man sells an article at 5% profit, Hf he had bought it at 5% less and sold it-for Re 1 less, he would have gained 10%, The cost price of the article is (a) Rs 200 (b) Rs 150 fe) Rs 250 (a) Rs 240 A shopkeeper sold a TV set for Rs 17,940 with a discount of 8% and gained 19.6%. If no discount is allowed, then what will be his gain per cent? (a) 25% 1b) 26.4% (©) 24.8% (a), None of these My friend collects antique stamps. She purchased ‘wo, but found that she needed to raise money urgently So she sold them for Rs 8.000 each. On one she made 20% and on the other she lost 20%, How much did she gain or lose in the entire transaction (a) No lossiprot. (hb) Rs 667 loss (o) Rs 667 profit, (a) None of these A trader wants 10% profit on the selling price of a product whereas his expenses amount to 15% on sales, What should his rate of mark up be on article of Rs 9? 30, 10, (a) 8.33% (©) 10% A man buys 6 dozen eggs for Rs 10.80, and 12 eggs rupee. Find his percentag 1 (ay 115 %egain 1 c) 9% gain fo) 97% L 9% loss OF Ian article is sold at 8% profit in stead of 8% loss would have brought Rs [2 more. Find out the cost price of the article fa) R875 (b) Rs 72 fc) R860 (a) Rs 70 Questions from Competitive Examinations (a) 20% (©) 30% On set pen at 5% loss and a book at 15% gain, Karim gains Rs 7. If he sells the pen at gain and the book at 10% gain, then he gains Rs 13. The actual price of the book is, (a) Rs 100. (b) Rs 80 (e) Rs 10 (a) Rs 400 By selling 33 metres of cloth. a shopkeeper gains the price of 11 metres of eloth. His gain per cent is, (a) 7% (b) 50% (©) 20% (a) 22% If 7% of the sale price of an article is equivalent to of is cost price and 9% of ts se price exceeds 10% of its cost price by Re I. then what is the cost price of the article? (a) Rs 400 (b) Rs 350 fe) Rs 300 (d) Rs 280 A cycle agent buys 30 bicycles, of which § are first grade and the rest are second grade, for Rs 3,150, Find at what price he must sell the first grade bicycles so that if he sells the second grade bicycles ar three quarters of the price, he may make aa profit of 40% on his outlay (a) Rs 200 (b) Rs (©) Rs 180 (d) Rs ‘The retail price of a water geyser is Rs 1,265. If the manufacturer gains 10% the wholesale dealer gains 15% and the retailer gains 25%, then the cost ff the product is (a) Rs 800 fc) Rs 700 240 210 (b) Rs 900 (d) Rs 600 aa You have either reached a page that is unavailable for viewing or reached your viewing limit for this book. aa You have either reached a page that is unavailable for viewing or reached your viewing limit for this book. aa You have either reached a page that is unavailable for viewing or reached your viewing limit for this book. 0.16 63 66. 67. 68. Quantative Apdoude (a) Rs 35.5 (©) Rs 76.5 (b) Rs 95.25 (a) None of these A shopkeeper sells a pair of sunglasses at a profit of 25%. If he had bougat it at 25% less and sold it for Rs 10 less, then he would have gained 40%, Determine the cost price of the pair of sunglasses, (a) Rs 50 (b) Rs 25 fe) Rs 75 (a) Rs 60 A video magazine distributor made 3,500 copies fof the march issue of the magazine at a cost of Rs 350,000. He gave $00 cassette free 10 some key video libraries. He also allowed a 25% discount fon the market price of the cassette and gave one extra cassette fiee with every 29 cassettes bought at a time, In this manner, he was able to sell all the K) cassettes that were produced. If the market price of a cassette was Rs 150, then what is his gain or Joss per cent for the march issue of video (a) 25% loss (©) 40% gain (b) 10% gain (4) 6.8% loss A trader marks his goods at such a price that he can deduct 15% for cash and yet make 20% profit, Find the marked price of an ilem which cost him Rs 90, (by Rs10s > u a) RSI 35— (a) a ca 598! uumit lent some money to Mohit at 5% per annum simple interest. Mohit lent the entire amount to 1 Biju on the same day at 84% per annum, In this transaction, after a year Mohit eamed a profit of Rs Find the sum of money lent by Sumit 10 Mohit (a) Rs 10,000 (b) Rs 9,000 (e) Rs 10,200 (a) None of these Am article is listed at Rs 65. A eusiomer bought this article for RS $6.16 and got two successive discounts of which one is 10%, The other rate of discount of this scheme that was allowed by the shopkeeper was (a) 3% (©) 6% (b) 4% (a) % Last year Mr Basu bought two scooters. This year he sold both of them for Rs 30,000 each, On one, he earned 20% profit, and on the other be made a 20% Joss. What was his net profit or loss? (a) He gained less than Rs 2,000 69. 70. n. 2 4. (b) He gained more than Rs 2,000 (c) He lost fess than Rs 2,000 (d)_ He lost more than Rs 2,000 A shopkeeper sells an article at 12% loss IPhe sells it for Rs.92.50 more then he gains 6%. What is the coxt price of the article? (@) Rs310 (e) RS375 (b) Rs 500 (d) Rs 600 A trader purchases a watch and a wall clock for Rs 390, He sells them making a profit of 10% on the ‘watch and 15% on the wall clock. He earsa profit of Rs 51.50. The difference between the original prices ofthe wall clock and the watch is equal to: (a R110 (b) Rs 100, fe) Rs8o (a) Rs 120 Padam purchased 30 kg of rice atthe rate of Rs 17.50 per kg and another 30 kg rice at a certain rate, He mixed the two and sold the entire quantity atthe rate of Rs 18.60 per kg snd made 20% overall profit. At what price per kg did he purchase the lot of another 30kg tice’ (a) Rs 14.50 (b) Rs 13.50 (c) Rs 12.50 (a) Rs 15.50 Vineet calculates his profit percentage on the selling price whereas Roshan calculates his profit on the cost price, They find that the difference of their profits is Rs 275, If the selling price of bath of them are the same, and Vineet gets 25% profit and Roshan gets 15% profit, then find their selling price. (a) Rs 2,100 (by Rs 2,300 fe) Rs 2,350 (a) Rs 2.250 A man sells two horses for Rs 1475. The cost price of | the first is equal to the selling price of the second. If the first is sold at 20% loss and the second at 25% zain, what is his total gain or loss (in rupees)? (a) Rs80 gain (b) Rs 60 gain (©) Rs 60 loss (d) Neither gain nor toss A Turner sold @ cow and an ox for Rs 800 and gota profit of 20% on the cow and 25% on the ox. If he sells the cow and the ox for Rs 820 and getsa profit of 25% on the cow and 20% on the Ox, the individual ‘cost price of the cow and the ox is (a) Rs 515.60, Rs 115.60 (Appros.) (b) Rs 531.50, Rs 135.50 (Approx.) (e) Rs $30.60, Rs 130.60 (Appros.) (a), Cannot be determined aa You have either reached a page that is unavailable for viewing or reached your viewing limit for this book. aa You have either reached a page that is unavailable for viewing or reached your viewing limit for this book. aa You have either reached a page that is unavailable for viewing or reached your viewing limit for this book. 9.20 § Quanuanve Aprsude yews CR =H tgs eto 10-2572 453 494.2 x 100 = cp, = EST — Rs 6.640 22 (a). Hore, SP SP. = Twn S ye 8 Using the formula CP. _ (SP, -S.Pp 100 xy weg GP eee 00 = p= 2ci00 = Rs 900 23 (b). Here, m = 17. Rs 1,882.75 Cost price of caleslater to Sita 100? OOF m OOM) “100 100% 1842.75 (100-17) (200-28) = 100 100%1882.73 _ pg M7x75 24 (a). We have, m= 10, =20, = = Rs 924 Cost price of 4 7 00? wit _ to 100 54000 {100— 10) (100+ 20) | 100 x 100 $4000 90120 2B (c). We have, m= 10, » = 20 Rs 50,000 Resultant profit % 29 (b). Here, m = 20, = Resultant profit or loss - (20-54 which represents profit as the sign is positive. 30 (c). Here, 5 + Oxeral toss =f 31 (c). Here, x = 20 Overall loss % 32 (a). Here x = 25 and y = ~20 Overall gain/loss % -| loix=y)#29 ly, 8 which represents loss because of the negative expression. 33 (6). Here, x = 20 and y = 40 = Over all gain "% | 100(x +.y)+ 2ay {100+ x)+ (100+) | . [!oaaneanre 2200} (10+ 20) + (100 +40) 7600 260 win 1B aa You have either reached a page that is unavailable for viewing or reached your viewing limit for this book. aa You have either reached a page that is unavailable for viewing or reached your viewing limit for this book. aa You have either reached a page that is unavailable for viewing or reached your viewing limit for this book. 924 Quantitaive Aptitude 23 (b). Profit would be maximum if books are bought for Sinee the initia profits sameasthe new so there Rs 200 and sold for Rs 42 is 0 increase in percentage. Profit = Rs (425 ~ 200) = Rs 225 27 (d). Marked price of the article = Rs 1,600 Profit of 8 books = Rs 225 x 8= Rs 1,800 Selling price = (100 ~ 10)% of (100 ~ 7%) 24(c). Lotthe C.P. of 1,000 gm of goods be Rs 1,000 of 1600 Marked price 1 1200 «1600 Selling price = Rs 1,200 * 0.9 =Rs 1,080 afiera discount of 10%) = ie 2 xc00 HK Rs 1,080 is the selling price of 900 g of goods (as 10 he cheats to the extent of 10% while selling) 122810 CP. of 900. =Rs 900 = “oxig = f00- Profit = Rs 180 F 120-+k (Profit) 180 oli % = he Profit = 182 x 100 = 20% 28(e), Profit %e= 795 = “Req Sate) S~ 1 900 ‘i 100 25(b). IfC.P.= 100, MP. = 130 Therefore, net profit%= 7575x100 = 10% 1 1 1 See gE RIO BRP 29a). 6 dozens egus cost = Rs 10.80 S.P. = 27,625 + 65 +22.75 = 115375 Since one dozen i rotten, he sells oly $ dozen Hence, profit = 15.375% at5 eggs per rupee. 26(c). Letthe C.P. of abicycle = Rs 100 Henee, S.P.=Rs 12 Now, since profit is 40% (12-10.8) this gain % x100 = U1 S.P.=Rs 240 Now, 7 bieyeles are being sold instead if 1 ieyele, but the sale price of new bicycle = Rs 120 “Therefore total sale price of new sale of bic 7 120=Rs 840 and the C P.=7 > 100 So the new profit = $40 - 700 = Rs 140 08 30 (a). Let the CP. ofthe article be » 108 -0.92¢ a 0.16 = outer Exercise 9.3 Questions from Competitive Examinations 4 (c). Let S.P. of x articles = Rs 100 = CP. of 20 articles C.P. of one articles = Rs Profit = 2: a SP. of one article= 6.25 = = 2 100 = 10 = SP. of vattcles = 625% S ox = 20% 200 625x~ 100 => x= 16 = x= 200 3 (a). Let the C.P. of the article be Rs x . he a 4(d). SP. = Rs 17.940 Discount = 8% SP. =x + Sof r= Rs 17940. 100 20 Marked Price = -—————~ If CP. would have been (x — 5% of 4), Le, a= ise “ey Gain = 19.6% Rs 13% and S.P, would have been Rs 211), eT cr = then gain % = 10 100+ 19.6 Rs 13.000 aa You have either reached a page that is unavailable for viewing or reached your viewing limit for this book. aa You have either reached a page that is unavailable for viewing or reached your viewing limit for this book. aa You have either reached a page that is unavailable for viewing or reached your viewing limit for this book. 928 43 (a). 44 (b). © Quantitative Aptinde This S.P- is arrived alter giving a discount of 20% on MP. 16x MP.= Tt means that article has bee above the cost price. Let the costof component A and Bbe Rs 30and Rs ‘50 respectively. Thea, cast of production = Rs G0 ‘+ 50 + 20), where Rs 20 contributes 10 the other expenses, assuming total production cost Rs 106. Since, profit is 20%, selling price = Rs 120. Now, new cost price of component 4 = Rs 39 New cost price of component B= Rs 61 New production cost (other expenses do not change) = (39 + 61 + 20) = Rs 120 Since new S.P. = 120 « 1.1 = 132 132-120 Henee, marked 100% New profit per cent = 2100 = 10% New cost of component 4 = 30 * 1.2 = Rs 36 Now east of component B ~ 50 0.88 ~ Rs 44 New production cest = Rs (36 + 44 + 20) = Rs 100 New selling price is same. Hence, profit = 120 — 100 = 20 oF 20% 45 (a). Let the C.P, be Rs 100, then S.P. ~ Rs 114.28 (Profit = 14.28%) 46 (0). 47 (a). This S.P. is arrived after giving a discount of 11.11% on marked price, hence, if marked price =x Then, x * 0.8889 = 114.28 = x = Rs 128.56, which is 28.56% more than the C.P. Cost price of @ +4 +5) = 12 kg of fits RS (G00 + 320-300} = R692. Pata poft of $08 — Re 1380 13H ns z *. S.P. of fruits per ke ns Let the price of 100 em of cloth be Rs 100, but he gets 120 em of cloth for Rs 100, Hence, his actual cost for tem ~ 122 — Rs 3 120 ‘ Now, instead of selling 100 cm, by cheating he sells 80.cm of cloth for the cost price of 100 ‘em of cloth. To calculate his profit, the cost price of 80 cm of cloth = exw Rs 66.66. Selling price of 80 em of cloth (actually 100 ‘em for the buyers) at & discount of 20% = 100 * 0.8 = Rs 80 80-666 6 Profit % = x10 = 20.01% oF 20% approximately 48 (b). 48 (b). 50 (c). 54 (a). 52 (c). 83 (c). 54 (a). In such a case where S.P. of two items is same and loss per cent and profit 1 per cent is also same, there is always a loss on such transaction, and itis given by loss per cent = UO" = 1% 100 Let the original cost price be Rs 100 Then, profit New ce. New profit 295 profit 56 ~ 355x100 = 70% CP. of the ratio at a gain of 12% = Reis Rs 1.378 CP. of the radi at alse of 12% = nl 0.88 CP. = Rs 3,125 SP. = Rs 3.080 los = Re (3125 ~ 3080) = Rs 1,750 + Total Towal is Given : S.P. of 12 marbles = Re 1, loss = 20% 1 SP. of 12 = Rs = Rs 1.25 CP. of 12 marbles = RST > 5 Now S.P. of 12 marbles at o gain of 20% = CP. 12> 1256 12 “Rs 15 It means in order to gain 20% he should sell 12 marbles for Rs 1.5. Or for Re | he should sell Z = 8 marbles. Lat the cost price be Rs 100, gain 3396 = Rs 33 Z S.P.= Rs 133, Let the marked price be Rs x. The S.P. of Rs 133 fs bean ete ater ved a coun 6 5% 08 reeked pre x <095 = Ry 133 8 * 0.95 Required increase = Rs 140 Rs 100 Rs 40 = x = Rs 140 CP, = Ro 153, gain 20% 2 P.= 153 « 12 = Rs 183.60 Lot the marked price be Rs x then x x 0.85 = 183.60 183.60 _ a a1 = eS oas Applying successive discounts of 10%, 12% and 15% on 100. aa You have either reached a page that is unavailable for viewing or reached your viewing limit for this book. aa You have either reached a page that is unavailable for viewing or reached your viewing limit for this book. aa You have either reached a page that is unavailable for viewing or reached your viewing limit for this book. 982 Quanutatve Aputude 5. A tradesman marks his goods at 25 per cent above cost price and allows discount of 12.5 per cent for ceash payment. What profit per cent does he make? 3 1 a) 92 b) 94 @% ©) 9% 5 7 © % (@) 95 6 Anarticle is at present being sold ata profit of 0 per cent, Ifthe cost of production rises by 20 per cent, and the selling price is inreased by 10 per cent, what will then be the profit per cent? 2 1 152% b) 192% @cis Dae © 12% (4) None of the above 7. Byselling ¢ watch ata profit of 10 per cent, a man got Rs 15 more thar halfits price. What isthe price of the watch? (@) 10 (b) 15 (c) 25 @) 5 BA shopkeeper gives three successive discounts of 22%, 17% and 12% on his goods. How much discount should his neighbour shopkeeper give on the same item sothat the discounts in both the cases will be same? (a) 39% (b) 41% (©) 43% (a) 45% 9 tradesman allows a discount of 15% on the marked price. How much above the C.P. must he mark his ‘goods to make a profit of 19 per cents (a) 25% (b) 34% (©) 40% (a) 46% 10. The CP, of four different qualities of sugars Rs 12, Rs 15, Rs 18, and Rs24 respectively. Ifthey are mixed in the proportion 4: 3:2: 1, then find the S.P. in ANSWERS Exercise 9.4 © © 4 0) 9%) 10.) a.) 17, @ 18. (6) @) 20. ©) Exercise 9.5 © R @ 4. (a) OO WOH me mE a. 2, 13, 14, 15, 13, rupees perkg in order to make a profit of 10 per cent (@) Rs 1550 (b) Rs17 © RsI8 (@ Rs24 There will be a 66.67% increase in the gain if awatch issold for Rs 1000 rather than at Rs ‘x-+ 120°, What is the cost price (c) of the watch? (@) Rs 820 (b) Rs 750 (©) Rs 800 (@) None of these Ankit purchased 120 tables at a price of Rs 110 per table. He sold 30 tables at a profit of Rs 12 per table and 75 tables ata profit of Rs 14 per table, Remaining tables were sold at loss of Rs 7 per table. What is the average profit per table? (@) Rs 10.04 (b) Rs 12.875 (©) Rs 10875 @ R843 ‘A merchant buying goods abroad at discount of 33 4 % of the catalogue price, has to pay import duty (0f 20% on the net cost of goods. If he realize « profit (025% on his outlay, what must he charge for anartcle priced Rs 228 in the catalogue? (@) 228 (b) 322 (©) Both (@) and (6) (d) Data insufficient ‘Aman would gain 20% by selling a chair for Rs 47.5 and would gain 15% by selling a table for Rs $7.5. He sells the chair for Rs 36; what is the least price for which he must sell the table to avoid any loss on tne two together? (@) Rs 502 (b) Rs55.8 (© R560 @) Rs 53.6 Manish sold a toy at 20% profit and eamed a profit of Rs 3. How much profit Gin rupees) would he eam if ie sells 15 similar toys at 30% profit? (a) Rs 45 (b) Rs 52.5 (© R675 (@ Rs70 Level I © 6d) 7 Oe) & ©) @ 4. @) 15, (6) 1 Level II @ 6. (b) ©) ) @ 4. @) 18) aa You have either reached a page that is unavailable for viewing or reached your viewing limit for this book. aa You have either reached a page that is unavailable for viewing or reached your viewing limit for this book. aa You have either reached a page that is unavailable for viewing or reached your viewing limit for this book. 14 Quanttative Aptitude Time taken by 4 and B, working together, to ‘complete the job laa eae =f « SC 7. If is times more efficient than B and is therefore able te finish a work in / days less than B, then (a) A and B, working together, can finish the 6 days work in days, Bol (b) A, working alone, can finish the work in t — days, kot kl (©) B, working alone, can finish the work in —— days Mustration 8 4 is thrice as good a workman as B and takes 10 days less to do a piece of work than B takes. Find the time in which B alone can complet the work. Solution Here, = 3 and /= 10 _ Time taken by B, working alone, to complete the work = Lays Rel 3x10 3 days, ie. 18 days SC 8. If A can complete ; part of work in X days, then beexX axd par or the work will be done in ays d Mlustration 9 4 can do 2th of « work in 12 days. In 4 how many days can he finish athe of the work? Solution Here, a=3, b= 4,X=12,¢-1andd = & ‘Therefore, number of days required to finish “th of the work ‘ SC 9. (a) There are two groups of people with same efficiency. In one M, persons can do HW’, works in Dy time and in the other M, persons can do W,, works in D, time. The relationship between the two groups is given by M,D,W,=M,D.W, (©) There are two groups of people with same cfficiency. In one M, persons can do W, works in D, time working f, hours a day and M, persons can do W¥, works in D, time working 1, hours a day. The relationship between the ‘two groups is given by MDytWy=MyDat,¥, Mustration 10 If 10 persons can complete 2/5th of a work in § days, then find the number of persons required to complete the remaining work in 12 days. Solution’ We have, M,= 10,17, M,=% W,=2,D)=2 BaD MDW = MDM, = xax Bow xire2 5 = M,=10 Wustration 11 If 10 persons can cut 20 trees in 3 days working 12 hours « day. Then, in how many days can 24 persons cut 32 trees working 4 hours a day? Solution We have,M,= 10, W,=20, Dy=3, 4, = 12 M,=24,W, MDytWy= MDa, = 10+ 312% 32=26*D,¥ 4» 20 = 6 days 3 SC 10, [Fa men and 6 women can do a piece of work in 7 days, then c men and d women can do the work in nab (st3} days Mustration 12 12 men or 15 women can do work in \4 days. In how many days, 7 men and 5 women would complete the work? Solution Here, a= 12, Required number of days 15,n= 14,027 and d=$ = ata ( 4x25) ayy, brad \1Sx7+12«5 168 3 Tp dys on 1S days aa You have either reached a page that is unavailable for viewing or reached your viewing limit for this book. aa You have either reached a page that is unavailable for viewing or reached your viewing limit for this book. aa You have either reached a page that is unavailable for viewing or reached your viewing limit for this book. aa You have either reached a page that is unavailable for viewing or reached your viewing limit for this book. aa You have either reached a page that is unavailable for viewing or reached your viewing limit for this book. aa You have either reached a page that is unavailable for viewing or reached your viewing limit for this book. aa You have either reached a page that is unavailable for viewing or reached your viewing limit for this book. 1012 © Quanusuve Apuude 32, 33. 34. 35. 36. 37. ©) 7Lhours @ 9 hours (@) 6 hours (© & hours 15 men take 21 days of 8 hours each to do a piece of work. How many days of 6 hours each would 21 women take, if3 women do as much work as 2 men? (@ 2 () 18 © 2% (@) 30 A certain job was assigned to a group of men to do in 20 days. But 12 men did not turn up for the job and the remaining men did the job in 32 days. The original number of men in the group was @ 32 (b) 34 © % @ 40 Aman is paid Rs 5 per hour for regular working day and Rs 8 per hour overtime. A regular working day has 8 hours and a regular week has 5 working days. the earns Rs 920 in 4 weeks, then what is the total number of hours that he worked assuming that he ‘was present on all regular working days? (2) 200 hours (b) 180 hours (175 hours (@) 160 hours Ram finishes a work in 7 days. Rahim finishes the same job in 8 days and Robert in 6 days. They take turns to finish the work. Ram worked on the first day, Rahim on the second day and Robert on the third and then again Ram and so on, Who was ‘working on the last day wien work got finished? @ Ram (b) Rebim (©) Robert (@) Rehim and Robert Ritu and Somaya are working on an assignment. Ritu takes 6 hours to type 32 pages on computer, while Somaya takes 5 hours for typing 40 pages. How much time will they take, working together on two different computers to type an assignment of 110 pages? (@ 8 hours (b) 7 hours 30 minutes (©) 8 hours 15 minutes (d) 8 hours 25 minutes Ifa person makes J jobs in M minutes, then how many jobs will he make in 2/3 hour? 2d 2M @ BL wy 2M © 3M Os7 9 ol, ma © oF @ FT AA boy read 3th of the book on one day and $n of the remainder on another dey. If there were 30 pages unread, then how many pages did the book contain? (@) 600 © 240 (b) 300 (d) None of these 38. 39, 40. aL a. 43 44, ‘A mill worker's basic pey for a 40-hour week is Rs 20. ‘Overtime is paid for at 25% above the basic rate. Ina certain week, he worked overtime and his total wage ‘was Rs 25, He therefore worked for atotal of (@ 40 hours (b) 42 hours (©) 48 hours (@ 50 hours ‘Three workers, werking all days, can do a work in 10 days, tut one of them having other employment can work only half time. In how many days the work can be finished? (@) 15 deys () 16 days (© 12 days (@ 125 days Sam, Bob and Kim can do a job alone in 15 days, 10 days and 30 days respectively. Sam is helped by Bob and Kim every third day. In how many days will the job be completed? (a) 9 days C) 8st days (© 8 days o Cs days Construction of @ road was entrusted to a civil engineer, He was to finish the work in 124 days for ‘hich he employed 120 workmen, Two-hirds ofthe work was completed in 64 days. How many workmen can be reduced now without affecting the completion of the work on time? @ % ) 64 © @ #4 ‘Two workers carned Rs 225. The first worked for 10 days and the second for 9 days. How mech did each of them get daly ifthe frst worker got Rs 15 more for working 5 days than the second worker fot for working 3 days? (a) Rs 11.70; Rs 12.00 (b) Rs 10.80; Rs 13.00 (© Rs 11.25;Rs 12.50 (d) Rs 12.60; Rs 11.00 When some technological improvements hed been ‘made, it took an hour less than before to manufacture a certain tool. The same time is now needed to manufacture 30 tools as was needed earlier to manufacture 24 tools. How much time does it require ‘now to make one tool? @ 5 hrs (© 4 hrs (© 3 hrs © 35 his ‘One man ean do as much work in one day as @ woman can de in 2 days. A child docs one-third the work in a day as a woman. If an estate owner hires 39 pairs of hands—men, women and children in the ratio 6 : 5: 2—and pays them in all Rs 1, 113 at the end of the day’s work, what must the daily wazes of a child be, if the wages are proportional to the amount of work done? aa You have either reached a page that is unavailable for viewing or reached your viewing limit for this book. aa You have either reached a page that is unavailable for viewing or reached your viewing limit for this book. aa You have either reached a page that is unavailable for viewing or reached your viewing limit for this book. 10.16 ® Quantitative Aptinde Exercise 10.1 4 (a). Here, X= 15 and ¥ = 12 = Working together, 10 men and 15 women will complete the work in days X4Y = BR ie, or, 62 anys 15412 30° 3 2 (b). Here, X= 30 and ¥ = 40, :. A and B working together can do the work in = days. var 30*40 120 gl days,ie, 2? or, 17 4 days Sag ey, PAL OM 3 (b). 4 can do the complete work in 5 * 3 {can do the complete work in 10 * = Here, X = 15 and ¥ = 25. 2 _ est, work oenab ei sip a 15 days. 25 days. work in MY says WoF 15x25 1 days, ie 2 or, 9 days 15425 rane 4 (6). Here, X= 6, ¥ = 12 and Z = 2 .. Working together, 4, # and C will complete the work in merece Eling ays ¥Z+20 61224 ae days, 6124 12% 24 +24 x6 24 3 ie, <> days or, 35 days 7 7 5 (b). Let B takes x days to do the work. Then, A takes, (r ~ 60) days to complete the work. Since ratio of work done by A and Bis 3: 1, ratio of time taken by 4 and B is 1 : 3 x-60 | We have, x3 {HINTS AND soLuTions } Level I = 3-60) =3 0, += 90 Time teken by B to finish the work = 90 days and time taken by 4 to finish the work 90 0 days. 3 A and B, working together, will complete the work days X+Y 30830 days, ice, on 22-1 days 90+30 a 2 6 (c). Let Ramesh takes x days to finish the work Then Maha ke Sand Sesh les 2 days to finish the same Work. . Ramesh, Mahesh and Suresh, working logser wil comple wor = 4 bays ¥ = A ays o. Ramesh takes 24 days, Mahesh takes = or 12 days and Suresh takes 3¥ or # days to finish the work. |. Let Gita take.x days to complete the work. Then, Sita takes 2x days to complete the same work. ©. Time taken by Rita to complete the work

Potrebbero piacerti anche